108
[files were scanned in and so could contain spelling/typo issues] Key To Database 1978 Education of women, 15 th to 18 th centuries 1979 Reign of Terror, French Revolution 1980 Witchcraft Scare 1981 Class Attitudes toward Industrialization 1982 Child-rearing in GB 16 th to 18 th c. 1983 Flemings and Walloons 1984 German Aircraft Industry 1985 Juvenile Crime and Treatment in GB 1986 Sudan Crisis (1884-1885) 1987 Literacy in Old Regime France 1988 Gin Act in G.B.; 18 th c. social history 1989 Women‟s Status/Suffrage in late 19 th /early 20 th c. 1990 Spanish Civil War 1991 Anti-slavery during the Enlightenment and French Revolution 1992 Pan-Slavism 1993 Renaissance Education 1994-1997 (not included in file b/c available for sale with answer keys and student essays from the college board) 1998 German Unity before 1848 1999-2003 (not included in file b/c available at the college board website for download)

AP Euro DBQ Database

Embed Size (px)

Citation preview

Page 1: AP Euro DBQ Database

[files were scanned in and so could contain spelling/typo issues]

Key To Database

1978 Education of women, 15th

to 18th

centuries

1979 Reign of Terror, French Revolution

1980 Witchcraft Scare

1981 Class Attitudes toward Industrialization

1982 Child-rearing in GB 16th

to 18th

c.

1983 Flemings and Walloons

1984 German Aircraft Industry

1985 Juvenile Crime and Treatment in GB

1986 Sudan Crisis (1884-1885)

1987 Literacy in Old Regime France

1988 Gin Act in G.B.; 18th

c. social history

1989 Women‟s Status/Suffrage in late 19th

/early 20th

c.

1990 Spanish Civil War

1991 Anti-slavery during the Enlightenment and French Revolution

1992 Pan-Slavism

1993 Renaissance Education

1994-1997 (not included in file b/c available for sale with answer keys and student essays from

the college board)

1998 German Unity before 1848

1999-2003 (not included in file b/c available at the college board website for download)

Page 2: AP Euro DBQ Database

1978 DBQ

Directions: The following question is based on the accompanying Document Groups A-C

This question is designed to test your ability to work with historical documents. You may refer to

historical facts and developments not mentioned in the documents and assess the reliability of the

documents as historical sources where relevant to your answer.

The Question: Discuss the extent to which early modern European Society encouraged

education for women. What criteria were used to evaluate women‟s education or its role, and

women‟s potential for learning? What evolution, if any, can be seen in attitudes toward education

for women from the Renaissance through the early eighteenth century?

Document Group A: THE RENAISSANCE

(1) “The Court Lady, must have not only the good judgment to recognize the kind of person

with whom she is speaking, but must have knowledge of many things, in order to entertain that

person graciously. . I say that women can understand all the things men can understand and that

the intellect of woman can penetrate wherever a man‟s can… there have always been women

who have undertaken wars and won glorious victories, governed kingdoms with the greatest

prudence and justice, and done all that men have done. As for the sciences, do you not remember

reading of many women who were learned in philosophy? Others who excelled in poetry?

Others who prosecuted, accused, and defended before judges with great eloquence?.....‟

Castiglione, The Courtier, Italy, 1528

(2)

Abbot: Distaff and spindle are the proper equipment for women.

Lady. Isn‟t it a wife‟s business to manage the household and rear the children?

Abbott: It is.

Lady: Do you think she can manage so big a job without wisdom?

Abbot: I suppose not.

Lady: But books s teach me this wisdom.

Abbot: I could put up with books, but not Latin ones.

Lady: Why not?

Abbot. Because that language isn‟t fit for women.

Lady: I want to know why.

Abbot: Because it does little to protect their chastity.

Erasmus, “The Abbot and the Learned Lady”, Switzerland, 1524

Page 3: AP Euro DBQ Database

(3)

“Study busies the whole soul…. It is not only a weapon against idleness but also a means of

impressing the best precepts upon a girl‟s mind and of leading her to virtue.

Erasmus, Christiani monii

Institution, Switzerland, 1526

(4)

All I can do is to beg our virtuous ladies to raise their minds somewhat above their distaffs and

spindles and try to prove to the world that if we were not made to command, still we should not

be disdained as companions in domestic and public matters by those who govern and command

obedience. Apart from the good name that our sex will acquire thereby , we shall have caused

men to devote more time and effort in the public good to virtuous studies for fear of seeing

themselves left behind by those over whom they have always claimed superiority in practically

everything ….. If there is ,anything to be recommended after honor and glory, anything to

incite us to study, it is the pleasure which study affords

A letter from the poet

Louise Labe to a friend,

France, 1555

(5)

I have just given you my opinion of the advantages that women may derive from a superior

education. However, I have nearly always found that such preparation turned out to be useless

for women of middling rank like yourselves…. There is the fact, moreover, that a dispropor-

tionate elevation of the mind is very apt to breed pride. I have seen two had effects issue from

this: (1) contempt for housekeeping, for poverty, and for a husband less clever than oneself, and

(2) discord. And so I conclude that 1 would be most reluctant to encourage girls to pursue book

learning unless they were princesses. obliged by their rank to assume the responsibilities,

knowledge, competence, administration, and authority of men.

Théodore Agrippa d‟Aubigné, writer and historian, to his daughters, France, circa 1590

(6)

“She has just passed her sixteenth birthday and shows such dignity and gentleness as are wonder-

ful at her age and in her rank, Her study of true religion and learning is most eager. Her mind has

no womanly weakness, her perseverance is equal to that of a man, and her memory long keeps

what it quickly picks up. She talks French and Italian as well as she does English, and has often

talked to me readily and well in Latin, moderately in Greek

Roger Ascham, describing his student, the future Queen Elizabeth 1, 1549

Page 4: AP Euro DBQ Database

Document Group B: REFORMATION AND CATHOLIC REFORMATION

(1)

“Men have broad shoulders and narrow hips, and accordingly they possess intelligence. Women

have narrow shoulders and broad hips. Women ought to stay at home; the way they were created

indicated this, for they have broad hips and a wide fundament to sit upon (to keep house and bear

and raise children).”

Martin Luther,

Table Talk, 1566

(2)

To learn essential doctrine... there is no need for the women or the artisan to take time out from

their work and read the Old and New Testament . . Then they‟ll want to dispute about it and give

their opinion . . and they can‟t help falling into error. Women must be silent in Church, as Saint

Paul says.

Edmond Auger, a

French Jesuit, 1566

(3) “If God has done the grace to some poor women to reveal to them by his Holy Scriptures

some good and holy thing, dare they not write about it, speak about it, and declare it one to the

other. . . . Is it not foolishly done to hide the talent that God has given us?”

Marie Dentière, a devout Calvinist, writing in “The

War and Deliverance of the City of Geneva,” 1539

Page 5: AP Euro DBQ Database

Document Group C: SEVENTEENTH AND EARLY EIGHTEENTH CENTURIES

(1)

„Whence true authority in men, though both

Not equal, as their sex not equal seem‟d,

For contemplation he and valor form‟d

For softness she, and sweet attractive Grace,

He for God only, she for God in him.

John Milton, Paradise

Lost, 1667

(2)

“I‟m speaking to you, sister ... I don‟t like all these useless books of yours. Apart from

the big Plutarch that keeps my neckbands pressed, you should burn them all. Get rid of this

fierce-looking telescope and all the rest of these gadgets…. Stop trying to find out what‟s

happening on the moon and mind what‟s going on in your own house where everything is upside

down. It‟s not decent, and there are plenty of reasons why it isn‟t, for a woman to study and

know so much. Teaching her children good principles, running her household, keeping an eye on

her servants, and managing her budget thriftily are all the study and philosophy she needs

Women today … want to write books and become authors. . . . In my house, they know all about

the moon and the pole star and about Venus, Saturn, and Mars, which are no concern of mine

and …. nobody knows how the pot is cooking…. Reasoning is the pursuit of everyone in my

household, and all their reasoning has driven out reason.”

From Moliere‟s play, The Learned Ladies, France, 1672

(3)

“The prodigious increase in the riches of the

Netherlands may be the wonder of all future generations. And vet the means whereby they

have thus advanced themselves are sufficiently obvious. . . . [the writer lists several, then] The

education of their children, as well daughters as sons: all which, be they of never so great quality

or estate, they always take care to bring up to write perfect good hands, and to have the full

knowledge and use of Arithmetic and Merchants-Accounts; the well understanding and practice

whereof doth strangely infuse into most that are the owners of that quality, of either sex, not only

an ability for commerce of all kinds but a strong aptitude, love and delight in it; and since the

women are as knowing therein as the men, it doth encourage their husbands to hold on in their

trades to their dying days, knowing the capacity of their wives to get in their estates, and carry on

their trades after their deaths.”

Sir Josiah Child, A New Discourse of Trade, England, 1668

Page 6: AP Euro DBQ Database

(4) “There is little point in girls of common extraction learning to read as well as young

ladies or being taught as fine a pronunciation or knowing what a period is, etc. It is the same with

writing. All they need is enough to keep their accounts and memoranda: you don‟t need to teach

them fine handwriting or talk to them of style: a little spelling will do. Arithmetic is different.

They need it. . . . Educate your middle-class girls in the middle-class way. . . . Don‟t even con-

template embellishing their minds. Teach them their domestic duties, obedience to the their

husbands, and how to look after their children and train their small staff, to go regularly to

church on Sundays and holidays, to be modest with customers, honest in business . . Tell them

that nothing is more displeasing to God and men than stepping out of one‟s social station—all

are ordained by Providence, and God resists our endeavors to be other than He intended us to be.

Teach them to be moderate and that the peasant must not try to ape the bourgeois nor the

bourgeois the gentleman.”

From Mme. de Maintenon, mistress and later wife of Louis XIV, to two women, both former

students at her school in St. Cyr, in charge of girls‟ secondary schools, 1713

Page 7: AP Euro DBQ Database

1979 DBQ Directions: The following question is based on the accompanying Document Groups A-G. Read and analyze the documents and answer the question. This question is designed to test your ability to work with historical documents. You may refer to historical facts and developments not mentioned in the documents and assess the reliability of the documents as historical sources where relevant to your answer. In constructing your answer DO NOT simply summarize or repeat the contents of the documents. The Question: Discuss the advantages and the disadvantages of the Terror as an instrument of

the French Revolution.

[Historical setting: In the summer of 1793, the radical Jacobins purged their moderate rivals from the National Convention and took over ~he government of revolutionary France. The country was in disarray. To the north and the east the Prussian and Austrian troops were pushing back the armies of France and threatening to invade the infant republic. The British navy threatened the coasts of France. In western France a war of secession was underway backed by various social groups including the peasantry. In Lyon, another secessionist movement opposed the authority of the radical government in Paris, and there was agitation in several cities in the south and southwest. The country was also in economic crisis. In August and September, the Committee of Public Safety, which governed the Republic, instituted a series of sweeping reforms to deal with the emergency situation. A Revolutionary Tribunal (court) was enlarged, and a law of suspects was passed that legalized local revolutionary committees. Troops were sent to deal with the secessionist areas. When the Terror ended n July, 1794, 14,000 people out of a total population of 25 million had been guillotined, shot, or drowned.)

Page 8: AP Euro DBQ Database

Document Group A: Statistical Data on the Terror Document 1: France 1793

Page 9: AP Euro DBQ Database
Page 10: AP Euro DBQ Database

Document Group B: Leaders of the Convention Look at the Terror (4) “Revolution is the war waged by liberty against its enemies; a constitution is that which crowns

the edifice of freedom once victory has been won and the nation is at peace. The revolution-ary government has to summon extraordinary activity to its aid precisely because it is at war. It is subjected to less binding and less uniform regulations . . . because it is compelled to deploy, swiftly and incessantly, new resources to meet new and pressing dangers. Under a constitutional government little more is required than to protect the individual against abuses by the state, whereas revolutionary government is obliged to defend the state itself against the factions that assail it from every quarter. To good citizens revolutionary govern-ment owes the full protection of the state; to the enemies of the people it owes only death. . It must adopt the general principles of ordinary government whenever these can be rigorously applied without endangering public liberty. Its force to repress must be commensurate with the audacity or treachery of those who conspire against it……”

Maximilien de Robespierre,

In a speech to the National Convention, December 25, 1793

(5) “Citizens, how could anyone delude himself that you are inhuman? . . . Since the month of

May last, our history is a lesson about the terrible extremities to which indulgence leads. In that period . . . Custine had abandoned Mainz, the Palatinate, and the banks of the Rhine; Calvados was in revolt; the Vendée was victorious, Lyon, Bordeaux, Marseille, and Toulon were in arms against the French people; our armies were being beaten in the Pvrenees and around Mont Blanc, you were being betrayed by everyone. . . . Yet the greatest of our misfortunes was a certain fear of the concentration of authority necessary to save the state.”

Louis Antoine de Saint-Just,

in a speech to the National

Convention, February 26, 1794

Document C: THE TERROR AS SEEN FROM THE MODERATE RIGHT

(6) “You want to remove all your enemies by means of the guillotine I Has there ever been

such great folly? Could you make a single man perish on the scaffold without making ten enemies for yourself from his family or his friends? Do you believe that these women, these old men, these weaklings, those egoists, these stragglers of the Revolution, whom you imprison, are really dangerous? Of your enemies there remain among you only the cowardly and the sick. The strong and the brave have emigrated. They have perished at Lyon or in the Vendée; the remainder (consisting of some rentiers and shopkeepers] do not merit your wrath.”

Camille Desmoulins, journalist and former ally of Robespierre, in le Vieux Cordelier, December 20, 1793

Page 11: AP Euro DBQ Database

Document Group D: Public Opinion and the Terror in Paris

The documents in this group are excerpted from reports to the government on public opinion. (7) “The majority of the citizens agreed in unanimously [sic] saying that the tribunals act well,

that they acquit the innocent and punish the guilty, although murmurs are heard among the

public at their judgments.”

January 28, 1794 (8) “Bitter complaints already expressed numberless times, were repeated today of the arrest

and imprisonment of citizens who are good patriots and are victims of ambition, cupidity,

jealousy, and, in short, every human passion.”

February 23, 1794

(9)

“On seeing peasants on the scaffold, people said, What, have these wretches allowed themselves to be corrupted? If they were nobles or rich people it would not be strange, their being counter-revolutionists, but in that class we should expect all to be patriots. „The law is just,‟ people remarked, „it strikes rich and poor indiscriminately.‟ The verdicts of the Revolutionary Tribunal are always applauded.”

March 2, 1794

(10) “The revolutionary committees are every day falling into discredit. You daily hear that they

consist of a number of intriguers, who plunder the nation and oppress citizens, It is a fact that there is no section in Paris which is not dissatisfied with its revolutionary committee or does not seriously desire to have them abolished.”

March 30, 1794

Page 12: AP Euro DBQ Database

Document E: A Suspect Appearing Before a Revolutionary Committee

Contemporary French engraving of a revolutionary committee by an unknown artist, probably an

opponent of the terror.

Page 13: AP Euro DBQ Database

Document F: The Terror in Lyon

(12) “The guillotine and the firing squad did justice to more than four hundred rebels. But a

new revolutionary commission has just been established, and in a few days the grape shot, fired by our cannoneers will have delivered to us, in one single moment, more than four thousand conspirators. . . . The Republic has need of a great example . . . whilst the thunderbolt, which must exterminate them in an instant, will carry terror into the departments where the seed of rebellion was sown, it is necessary that the flames from their devastated dens proclaim tar and wide the punishment that is destined for those who try to imitate them.”

A letter from General Ronsin, a leader of the Revolutionary Army that subdued Lyon. December 17, 1793

Document Group G: REACTION BY BRITISH LEADERS

(13)

“What a pity that a people [the French] capable of such Incredible energy, should he guilty or rather be governed by those who are guilty of such unheard of crimes and cruelties.”

Charles James Fox, reformist member of Parliament and sympathizer with the French Revolution, in a speech to Parliament, November 7, 1793

(14) “But it has been urged, that the French have distinguished themselves in the field; nor

will it be denied that, independently of any other circumstance, the spirit of a people called forth by the impulse which acts so strongly in such a situation, may have the effect to make them brave in the moment of action. But their efforts are merely the result of a system of restraint and oppression, the most terrible and gigantic, that has, perhaps, ever existed. They are compelled into the field by the terror of the guillotine—they are supported there only by those resources which their desperate situation affords; and, in these circumstances, what can be the dependence on the steadiness of their operations, or what rational prospect can there be of the permanence of their exertions?

William Pitt, British Prime Minister, in a speech to Parliament, January 21, 1794

Page 14: AP Euro DBQ Database

The Witch DBQ 1980

Directions: The following question is based on the accompanying Document Groups A-D. You

will have 60 minutes to read and analyze the documents and answer the question. You may make

notes and plan your answer on the green insert or on the printed pages of the pink free-response

booklet. Write your answer to the question on the lined pages of the free-response booklet.

This question is designed to test your ability to work with historical documents. You may refer to

historical facts and developments not mentioned in the documents and assess the reliability of the

documents as historical sources where relevant to your answer. In constructing your answer DO

NOT simply summarize, paraphrase, or repeat the contents of the documents; instead use the

documents in a historical context and draw conclusions from them. Construct a coherent essay

that integrates the analysis of documents into a treatment of the topic.

The Question

Using the following documents, identify and analyze at least three major reasons for the

persecution of individuals as witches in Europe from the late fifteenth through the seventeenth

centuries.

[Historical setting: The witch craze lasted from about 1480 to 1700. This was the period of the

Protestant Reformation, the Catholic Reformation, the Scientific Revolution, and the

consolidation of national governments. Witches were persecuted in most of Europe, but the trials

were concentrated in southwestern Germany, Switzerland, England, Scotland, Poland, and parts

of France. The total number of accused witches who were tried exceeded 100,000. Torture was

used to extract confessions in many areas, but not in others; in England the trials were generally

conducted without the use of torture.]

Page 15: AP Euro DBQ Database

Document Group A: THE TESTIMONY OF ACCUSED WITCHES AND EYEWITNESSES

(1) “Walpurga Hausmannin . . . has, upon kindly questioning and also torture‟ . . . confessed her

witchcraft and admitted the following. When . . . she had become a widow, she cut corn for Hans

Schlumperger…. Him she enticed with lewd speeches and gestures and they convened that they

should . . . meet in her . . . dwelling, there to indulge in lustful intercourse …. [ But] it was not the

said bondsman who appeared unto her, but the Evil One [the Devil] in the latter‟s guise …. He

made her many promises to help her in her poverty and need, wherefore she surrendered herself to

him body and soul …. For food she often had a good roast or an innocent child, which was also

roasted, or a suckling pig …. [The Evil One] also compelled her to do away with and to kill young

infants at birth …. This she did as follows …. A child of the Governor here . . . she had so infected

with her salve that he died within three days …. Three years ago she had sucked out the blood of

[citizen] Kung‟s child, a twin, so that it died…. She had also rubbed a salve on a beautiful son of

the . . . Chancellor, . . . this child had lovely faire hair and she had given him a hobby horse so that

he might ride on it till he lost his senses. He died likewise….”

Testimony of a licensed midwife at Dillingen, Germany, burned 1587

(2) "This movement was promoted by many in office, who hoped for wealth from the

persecution. And, so, from court to court throughout the towns and villages of all the

diocese, scurried special accusers, Inquisitors, . . . dragging to trial and torture human

beings of both sexes and burning them in great numbers .... Nor were spared even the

leading men of the city of Trier. For the Judge with two Burgomasters, several Councillors

and Associate Judges, canons of sundry collegiate churches . . . were swept away in this

ruin .... Meanwhile notaries, copyists, and innkeepers grew rich. The executioner rode a

blooded horse, like a noble of the court, and went clad in gold and silver; his wife vied

noble dames in the richness of her array. The children of those convicted and punished were

sent into exile; their goods were confiscated. "

The Canon Linden, eyewitness to persecutions in Trier, Germany, 1592

Page 16: AP Euro DBQ Database

(3) "Presently he cryeth out of some poor Innocent neighbor that he

or she hath bewitched him. For, saith he, such an old man or woman

came lately to my door and desired some relief, and I denied it, and

God forgive me, my heart did rise against her . . . and presently my

child, my wife, myself, my horse, my cow, my sheep, my sow, my

hog, my dog, my cat, or somewhat, was thus and thus handled in

such a strange manner, as I dare swear she is a witch, or else how

should these things be"

Thomas Ady, describing the feelings of an English householder,

circa 1650

(4) "There is one Alice Prabury in our parish that useth herself

suspiciously in the likelihood of a witch, taking upon her not only to

help Christian people of diseases strangely happened, but also horses

and all other beasts. She taketh upon her to help by the way of

charming, and in such ways that she will tell nobody her sayings. "

Report of Churchwardens in Gloucestershire, England, 1563

(5) Some call me witch,

And being ignorant of my self, they go

About to teach me how to be one; urging,

That my bad tongue (by their bad language made so)

Forespeaks* their cattle, cloth bewitch their corn

Themselves, their servants, and their babes at nurse.

This they enforce upon me; and in part

Make me to credit it.

*makes prophesies or predictions against

'The Witch of Edmonton," a poem written in 1621

Page 17: AP Euro DBQ Database

(6) "It is seldom that a poor old wretch is brought to trial . . . but

there is, at the heels of her, a popular rage that could little less than

demand her to be put to death; and if a judge is so clear and open as

to declare himself against the impious vulgar opinion, that the devil

himself has power to torment and kill innocent children, or that he is

pleased to divert himself with the good people's cheese, butter, pigs

and geese, . . . cry, this Judge hath no religion, for he doth not

believe in witches."

Roger North, brother of the Chief Justice in Exeter, England in 1682

(7) "Innocent have I come into prison, Innocent have I been

tortured, Innocent must I die. For whoever comes into the witch

prison must become a witch or be tortured until he invents something

out of his head and-God pity him- bethinks him of something ....

When at test (the executioner) led me back into the prison he said to

me, 'Sir, I beg you, for God's sake confess something, whether it be

true or not. Invent something, for you cannot endure the torture

which you will be put to; and, even if you bear it all, yet you will not

escape, not even if you are an earl, but one torture will follow

another until you say you are a witch . . . as you may see by all their

trials, for one is just like another. . . . ' Dear child, keep this letter

secret so that people do not find it.... Good night, for your father

Johannes Junius will never see you more."

Letter of Johannes Junius, the mayor of Bamberg, Germany, to his

daughter 1628

GROUP B: RELIGIOUS OPINIONS

(1) "As for the question, why a greater number of witches is found

in the fragile feminine sex than among men . . . the first is, that they

are more credulous.... The second reason is, that women are naturally

more impressionable, and more" ready to receive the influence of a

disembodied spirit; and that when they use this quality well they are

very good, but when they use it ill they are very evil.... But the

natural reason is that she is more carnal than a man.... And it should

be noted that there was a defect in the formation of the first woman,

since she was formed from a bent rib, that is, a rib of the breast,

which is bent as it were in a contrary direction to a man... And since

through this defect she is an imperfect animal, she always deceives.

Kramer and Sprenger, The Hammer of Witches, a handbook used by

the Inquisition, written in 1484 by two Dominican monks

Page 18: AP Euro DBQ Database

(2) "It has recently come to our ears, not without great pain to us,

that . . . many persons of both sexes, heedless of their own salvation

and forsaking the Catholic faith, give themselves over to devils male

and female.... We therefore, desiring, as is our duty . . . to remove all

impediments by which . . . the . . . inquisitors are hindered in the

exercise of their office . . . do hereby decree, by virtue of our

apostolic authority, that it shall be permitted to the . . . inquisitors . . .

to exercise their office of Inquisition and to proceed to the

correction, imprisonment, and punishment of the aforesaid persons

for their said offences and crime...

Pope Innocent VIII, "The Witch Bull." 1484

( 3) ". . . sorcerers or witches are the Devil's whores who steal milk,

raise storms, ride on goats or broomsticks, lame or maim people,

torture babies in their cradles, change things into different shapes so

that a human being seems to be a cow or an ox, and force people into

love and immorality . . . not that the Devil is unable to do these

things by himself without sorcerers, for he is lord of the world yet he

will not act without human help."

Martin Luther, preaching in 1522

(4)"Moreover, in order that we may be aroused and exhorted . . .

Scripture makes known that there are not one, not two, nor a few

foes, but great armies, which wage war against us. For Mary

Magdalene is said to have been freed from seven demons by which

she was possessed [Mark 16:9; Luke 8:2], and Christ bears witness

that usually after a demon has once been cast out, if you make room

for him again, he will . . . return to his empty possession [Matt.

12:43-45]. Indeed, a whole legion is said to have assailed one man

[Luke 8:30]. We are therefore taught by these examples that we have

to wage war against an infinite number of enemies....

John Calvin, Institutes of the Christian Religion, 1536

Page 19: AP Euro DBQ Database

(5) "I suffered terribly from fear of Hell and the devils, whom I

thought I saw [everywhere] . . . and sometimes with great rolling

flaming eyes like saucers, having sparkling firebrands in one of thelr

hands, and with the other reaching at me to tear me away to

torments. Oh the leaps that I have made, the fright that I have had,

the fears that I was in."

From the diary of a young Protestant boy, late 16th century, E.

Rogers, Some Account of the Life and Opinion of a Fifth-Monarchy

Man, 1867

DOCUMENT GROUP C: SCIENTIFIC OPINIONS

(1) The bodies of aged persons are impure, which, when they

[become diseased with malice, they use their very breath and their

sight, being apt for contagion, and by the Devil whetted for such

purpose, to the vexation and destruction of others. For if they which

are troubled with the disease of the eyes called opthalmia do infect

others that look earnestly upon them, is it any marvel that these

wicked creatures, having both bodies and minds in a higher degree

corrupted, should work both these and greater mischiefs?"

W. Fulbecke, A Parallele or Conference of the Civil Law, the Canon

Law and the Common Law, 1618

(2) ". . . that childish old hags called witches can do anything to

harm men or animals . . . I fight with natural reason.... My object is

also medical, in that I have to show that those illnesses, whose

origins are attributed to witches, come from natural causes.... Since

witches are usually old women of melancholic nature and small

brains [women who get easily depressed and have little trust in God],

there is no doubt that the Devil easily affects and deceives their

minds by illusions and apparitions that so bewilder them that they

confess to actions that they are very far from having committed....

From consideration of their age and sex, Christians should be less

ready to throw these poor mindless old women into dark, black,

stinking prisons unfit for humans and inhabited by evil spirits that

torment the prisoners."

Johan Wier (a Belgian physician), De Praestigiis Daemonum, 1563

Page 20: AP Euro DBQ Database

Document Group D:

WITCHCRAFT STATISTICS DRAWN FROM CONTEMPORARY COURT RECORDS

(1) OCCUPATIONS OF THE HUSBANDS OF THOSE

ACCUSED OF WITCHCRAFT IN AN ENGLISH REGION, 1546-

1680

OCCUPATIONS OF

HUSBANDS

NUMBER RECORDED, 1546-

1680

Laborer 23

Farmer 11

Tailor 4

Yoeman 4

Mason 2

Sailor 2

Beer Brewer 1

Shoemaker 1

Weaver 1

Gentleman 0

From Alan Macfarlane, Witchcraft in Tudor and Stuart England: A

Regional and Comparative Study, 1970

(2)MALES AND FEMALES EXECUTED IN SOUTHWESTERN

GERMANY, SWITZERLAND, AND SELECTED PARTS OF

FRANCE

AREA MALES FEMALES

Southwestern

Germany 238 (18%) 1,050 (82%)

Switzerland &

Selected Parts of

France

305 (22%) 1,060 (78%)

From Alan Macfarland, Witchcraft in Tudor and Stuart England: a

Regional and Comparative Study, 1970

Page 21: AP Euro DBQ Database

(3)AGE OF SUSPECTED WITCHES

DATES SIZE OF

SAMPLE MEDIAN AGE

Basel 1609-1617 10 60 (1 under age

50)

Fribourg,

Germany 1607-1683 9

60 (2 under age

50)

Geneva 1537-1662 95 60 (24 under 50)

Essex, England 1645 15 60 (2 under 50)

Deot ofthe

Nord, Fr. 1542-1679 39 55 (14 under 50)

From H.C. Eric Midelfort, Witch Hunting in Southwestern Germany,

15652-1684: the Social and Intellectual Foundation, 1972

Page 22: AP Euro DBQ Database

1981 DBQ

Directions: The following question is based on the accompanying Document Groups A and B. You will have 60 minutes to read and analyze the documents and answer the question. You may make notes and plan your answer on the green insert or on the printed pages of the pink free-response booklet. Write your answer to the question on the lined pages of the pink essay booklet. This question is designed to test your ability to work with historical documents. You may refer to historical facts and developments not mentioned in the documents and assess the reliability of the documents as historical sources where relevant to your answer. In constructing your answer DO NOT simply summarize, paraphrase, or repeat the contents of the documents; instead use the documents in the historical context of the question and draw conclusions from them. Construct a coherent essay that integrates the analysis of documents into a treatment of the topic.

The Question: Compare middle-class and working-class attitudes toward work and its effect on the worker in nineteenth-century Western Europe. Did any attitudes cross social class lines?

Historical setting: The following documents are drawn from England, France, Belgium, and Germany during the first century of industrialization (1801-1910), when most workers and employers had at most only a generation or two of experience with the organization and technology of factory industry. The documents suggest, then, the attitudes toward work produced by contact with the early stages of industrialization. Workers and their middle-class employers generally had opinions on what work should be and the way in which these ideals compared to work as they actually experienced it.

Document Group A: Middle-Class Observations (1) A Sunday school hymn composed for the English Sunday school movement, which was largely

guided by members of the middle class:

How doth the busy little Bee Improve each shining hour And gather honey all the day From every opening flower.

In works of labour or of skill I would be busy too; For Satan finds some mischief still For idle hands to do.

From Isaac Watts, Divine and Moral Songs for Children, 1869

Page 23: AP Euro DBQ Database

(2) A report on Manchester textile workers by a medical doctor:

“Prolonged and exhausting labour, continued from day to day, and from year to year, is not calculated to develop the intellectual or moral faculties of man. The dull routine of a ceaseless drudgery, in which the same mechanical process is incessantly repeated, resembles the torment of Sisyphus—toil, like the rock, recoils perpetually on the wearied operative. The mind gathers neither stores nor strength from the constant extension and retraction of the same muscles. The intellect slumbers in supine inertness; . . . To condemn man to such severity of toil is, in some measure, to cultivate in him the habits of an animal

From James P. Kay, The Moral and Physical Conditions of the Working Classes Employed in Cotton Manufacture in Manchester, 1832

(3) Another comment on Manchester textile workers by a manufacturer:

“I have visited many factories, both in Manchester and in the surrounding districts, and I never saw children in ill-humour. They seemed to be always cheerful and alert, taking pleasure in the light play of their muscles—enjoying the mobility natural to their age. The scene of industry, so far from exciting sad emotions in my mind, was always exhilarating . . . . The work of these lively elves seemed to resemble a sport, in which habit gave them a pleasing dexterity. Conscious of their skill, they were delighted to show it off to any stranger.”

From Andrew Ure, Philosophy of Manufactures, (1835)

(4) Report by the owner of a textile factory in Lille, France:

“It is simply false to equate the hours of work in our factories with arduous work. My workers, for example, in principle put in ninety hours a week, but I am lucky to get seventy -two hours of work from them. They seize on any occasion to wander around the factory or even walk outside, and to chatter with each other. Sometimes I think that they do not know what work is, and can be made to work only against their will. Really, they are like children, but I wish we could get them to work as hard as our own schoolboys work. I will admit only that my second generation of workers, who grew up in the factory, are somewhat more amenable

From Archives Nationales de France, F124705, “Report of Barrois,” 1837

Page 24: AP Euro DBQ Database

(5) Management report filed in 1888 by a Ruhr coal-mining company:

“Another very disturbing and damaging factor is the high turnover of workers, which, along with absenteeism, is always a problem. Workers change jobs for various reasons, but in part simply to get some time off from work. They show little concern for main-taining their skill or productivity, for they believe that they are pushed to produce more than men ought to produce in any event, and that if they work too hard the company will simply cut their pay, to get the profit, or reduce the number of jobs and so throw many workers into unemployment. They talk of the old days, when they were not driven so hard. The same feeling causes some of the resistance to overtime shifts, though of course other workers are eager for such shifts to earn extra money.

From Gerhard Aldelmann, Quellensammlung zur Geschichte der sozialen Betriebsverfasung: Ruhrindustrie (Collected Sources on the History of the Social Management —Worker Agreements in Industry in the Ruhr Area), Reprinted Bonn 1965

(6) A personal letter written in Great Britain in 1820:

“The unhappy dislocation which has taken place between the Employer and those in his employment is owing to the steam engine. When the machinery was driven by water, the manufacturer had to seek out some spot where he could obtain a suitable fall of water, and then his workmen formed the inhabitants of a village around him, and he necessarily bestowed some attention, less or more, on their morals and on their necessities, had knowledge of their persons and characters, and exercised a healthy influence as over men depending on and intimately connected with him and his prospects. This is now quite changed; the manufacturers are transferred to great towns, where a man may assemble five hundred workmen one week and dismiss them the next, without having any further connection with them than to receive a week‟s work for a week‟s wages, nor any further solicitude about their future fate than if they were so many old shuttles

From Walter Scott, Familiar Letters, 1894

Document Group B: Observations by Workers

(7) Impressions of a Manchester spinner, 1818:

“Locked up in factories eight stories high, [the worker] has no relaxation till the ponderous engine stops, and then he goes home to get refreshed for the next day; no time for sweet association with his family; they are all alike fatigued and exhausted. This is no over-drawn picture: it is literally true . . . . When the spinning of cotton was in its infancy, there was work for all, and at a proper pace, and in the community of family and friends. This was before those terrible machines for superseding the necessity of human labour, called steam engines, came into use . . . and workmen lost their power over their labor.”

From Black Dwarf, 1818

Page 25: AP Euro DBQ Database

(8) Recollection of a child laborer in a British textile mill:

“For several years after I began to work in the mill, the hours of labour at our works did not exceed ten in the day, winter and summer; and even with the labour of those hours, I shall never forget the fatigue I often felt before the day ended, and the anxiety of us all to be relieved from the unvarying and irksome toil we had gone through before we could obtain relief by such play and amusement as we resorted to when liberated from our work. I allude to this fact because it is not uncommon for persons to infer that, because the children who work in factories are seen to play like other children when they have time to do so, the labour is, therefore, light, and does not fatigue them. The reverse of this conclusion I know to be the truth. I know the effect that ten hours labour had on myself

From John Fielden, The Curse of the Factory System, 1836

(9) A miner relating his experiences working in a German coal pit early in the twentieth

century:

“The work is becoming increasingly mechanical. No more incentive, no more haste, we muddle along wearily, we are worn out and mindless. My forehead burns like fire. As a consequence of the anemia from which I suffer, I occasionally experience a slight dizzy spell. But in my head it rages and paralyzes me beyond control or without my being able to think. When it becomes unbearable I stop my slow, energyless working. I then sit on the side wall of the mountain in order to slurp the last remaining coffee . . . . And that is not all; the spirit too, the conscience of the individual, degenerates. And one drudge, grown vacuous through his work, is put beside another one, and another one and finally this „modern‟ circle has closed in on the entire working force.”

From Adolf Levenstein, Aus

der Tiefe, Arbeiter Briefe

(From the Depths: Workers

Letters), 1905

(10) Comments of an early twentieth-century Belgian coal miner:

“As if the bosses weren‟t enough, most of my workmates and I feel the pressure of some of our own fellows. A few are eager beaver types. They‟ll always try to fill the most wagons with coal or work overtime when we‟re asked. Some of them have special expenses of course, like a sick child and then that‟s all right, but some just seem to want to show the bosses how good they are, and make more money, and they don‟t care how they make us look or what they do to our jobs. They are to-morrow‟s foremen, or worse. Then we have the Flemish peasants, fresh from the countryside. They have no skill, they‟re dangerous to work with, but they just plod along like animals. And they call us bums for taking our breaks and a day off to play now and then. They too will do anything they‟re told, work any hours, as if they were still on the land. Most of them hope to go back in any event, and they don‟t care about a sensible life here.”

From Jules Lekeu, A Travers le Centre: Croquis et moeurs; enquete ouvrière et industrielle (Through the Center: Sketches and Customs; An Investigation of Labor and Industry), 1907

Page 26: AP Euro DBQ Database

1982 DBQ

Directions: The following question is based on the accompanying Document Groups A-C. You will have 60 minutes to read and to analyze the documents and answer the question. (Some of the documents have been edited for the purpose of this exercise.) It is suggested that you take five minutes to plan and outline your answer on the green insert or on the printed pages of the pink free-response booklet. Write your answer on the lined pages of the free-response booklet. This question is designed to test your ability to work with historical documents. You may refer to historical facts and developments not mentioned in the documents and assess the reliability of the documents as historical sources where relevant to your answer. In constructing your answer DO NOT simply summarize, paraphrase, or repeat the contents of the documents; instead use the documents in the historical context of the question and draw conclusions from them. Construct a coherent essay that integrates the analysis of documents into a treatment of the topic.

The Question: Analyze continuities and changes in methods of child-rearing among the English upper classes from the sixteenth through the eighteenth centuries. How did adult views of children shape adult practices toward their children?

Document Group A: Perceptions of Children (1) “The young child which lieth in the cradle is both wayward and impulsive; and though his

body be but small, yet he hath a wrong-doing heart, and is altogether inclined to evil. If this spark be allowed to increase, it will rage over and burn down the whole house. For we are changed and become good not by birth but by education. Therefore parents must be wary and circumspect. They must correct and sharply reprove their children for saying or doing ill.”

Robert Cleaver, a Calvinist minister writing in Godly Form of Household Government, 1598

(2) “A child is man in a small letter yet the best copy of Adam before he tasted of Eve or the apple. His soul is yet a blank paper unscribbled with observations of the world, whereas in time it becomes a blurred notebook. He kisses and loves all, and, when the sting of the rod is past, he smiles on his beater. He is purely happy because he knows no evil.”

John Earle, an Anglican minister writing in Microcosmographie, 1628

(3) “I can no longer contain my joy for our good fortune to have that dear child [Ralph Verney,

seven years old] you have left in our hands. He is in perfect health and seems contented, and the sweetness of his temper and vivacity of spirit, joined with the innocence of his age renders him the delight of all about him. He has the liberty of ranging the garden and the fields and on church days has the sole responsibility for the ringing of the bell.”

Letter of an Anglican rector to the Verney family, 1721

Page 27: AP Euro DBQ Database

(4)

Page 28: AP Euro DBQ Database

Document Group B: Infant Care---Nursing and Clothing

Nursing:

(5) “Because it hath pleased God to bless me with many children, I thought good to open my mind

concerning the duty of nursing (breastfeeding) due by mothers to their own children. Women ought to nurse their own children rather than hire a wet nurse*. They are very willful or ignorant if they question it. I know I should have done it, but partly I was overruled by another‟s authority and partly deceived by some ill counsel and partly I did not consider my duty in this motherly office. Now I try to regain my peace by preventing many Christian mothers from sinning in the same way. *a woman who cares for and suckles an infant not her own

Elizabeth Clinton, The Countess of Lincoln‟s Nursery, 1622

(6) “Lady Fitzwilliam and her newborn son are perfectly well. She has breastfed him several times. I

think in my life I never saw so happy a creature as Lord Fitzwilliam. He really is almost out of his senses with joy, and can see, think and talk of nothing but his child.”

Letter from Lady Duncannon to Georgiana, Lady Spencer, May 6, 1786

(7) “The first of the parental duties which nature points out to the mother is to nurse her own off-

spring. To transfer to a stranger, as modern example dictates, the task of nursing your child, when your health and strength are adequate to the undertaking is to exhibit a most shameful degree of selfishness and unnatural insensibility.”

Thomas Gisborne, author of a popular handbook entitled An Enquiry into the Duties of the Female Sex, 1799

Clothing:

(8) “By experience we can see that mothers in swaddling* their little ones do lay their limbs right, each in its place: likewise if a child be lefthanded, they scold him: yea, sometimes they bind it up, or otherwise restrict the use of it.” *to wrap narrow strips of cloth around an infant

Robert Cleaver, Godly Form of Household Government, 1598

Page 29: AP Euro DBQ Database

(9) “In the past, parents wanting to make a show of the infant as soon as it was born, were

ambitious to have as much finery heaped upon it as possible, and the poor child, as soon as it came into the world, had as many rollers and wrappers applied to its body, as if every bone had been fractured in the birth; while these were often so tight, as not only to irritate and wound its tender frame, but even to obstruct the motion of the heart, lungs, and other organs necessary for life. In several parts of Britain, the practice of rolling children with so many bandages is now, in some measure, laid aside.”

William Buchan, M. D., Domestic Medicine; or the Family Physician, 1772

Document Group C: Modes of Discipline

(10) “When I am in the presence either of father or mother, whether I speak, keep silent, sit,

stand or go, eat, drink, be merry or sad, be sewing, playing, dancing, or doing anything else, I must do it, as it were, in such weight, measure and number, even so perfectly as God made the world, or else I am so sharply taunted, so cruelly threatened, yea presently sometimes with pinches and bobs*, and other ways I will not name for the honor I bear them, that I think myself in Hell. .

*blows with the fist Account by Lady Jane Grey of her childhood experiences in the 1530‟s, later related to her tutor Roger Ascham

(11) “I myself have known some furious parents who have vied to strike and buffet their

children about the face and head, and to lay upon them like malt-sacks with cudgels, staves, pitchfork or fire shovel or whatsoever came into their hands; yea, and very often have cast them on the ground and spurned and kicked them like dogs. If parents would use only the rod, then they would have the means to rejoice and become parents of wise and worthy children.”

Bartholemew Batty, The Christian Man‟s Closet, 1581

Page 30: AP Euro DBQ Database

(12) “You must begin early to make your children love you so that they will obey you. You must

deny them as seldom as you can. When there is no avoiding it, you must deny them gently, flattering away their ill-humors and taking the next opportunity of pleasing them in some other things before they either ask or look for it. Let them be more in awe of your kindness than of your power, and above all, take heed of supporting a favorite child in its impertinence, which will give right to the rest of claiming the same privilege.”

Sir George Savile, first Marquis of Halifax,

The Lady‟s New-Years Gift; or Advice to a Daughter, 1687

(13) “Lord Holland‟s education of [his son, Charles] was a system of the most unlimited

indulgence of every passion, whim, and caprice. A great dinner was given at Holland House to all the foreign ministers. The children came in at the dessert. Charles, then in petticoats, spying a large bowl of cream in the middle of the table, had a desire to get into it. Lord Holland insisted he should be gratified and in spite of Lady Holland‟s protests had it placed on the floor for the child to jump in and splash about at his pleasure.”

Notes by Lady Louisa Stuart on George Selwyn and His Contem-poraries, 1760

(14) The object which I have in view is to make my son an English gentleman. I am convinced

that his mind may be moulded into any form, by the mere influence and persuasion of generous, pathetic, or rational motives. With respect to discipline, I absolutely forbid the use of blows. There are a thousand better ways of restraining and correcting a child, such, for instance, as confinement, neglect, fasting, being kept from play, etc. On the same principle, I would not have him called a dunce, or stupid, or blockhead. That way of degrading can produce no good effect.”

Letter of Sir Philip Francis to his son‟s tutor, February 17, 1774

Page 31: AP Euro DBQ Database

1983 DBQ

Directions: The following question is based on the accompanying Documents 1-12. (Some of the

documents have been edited for the purpose of this exercise.) Write your answer on the lined

pages of the pink essay booklet.

This question is designed to test your ability to work with historical documents. You may refer to

historical facts and developments not mentioned in the documents. Construct a coherent essay

that integrates the analysis of documents into a treatment of the topic.

The Question: Identify the major features that distinguished Flemings from Walloons in

Belgium in the nineteenth and early twentieth centuries. What political, economic, and social

tensions developed between the two groups?

Historical setting: In 1830 a new independent nation-state, Belgium, was created out of the

southern provinces of the kingdom of the Netherlands. Belgium's liberal constitutional monarchy

united population groups of different cultural backgrounds. Until the First World War, this small

mineral-rich state, long a leader in European commerce, shared many economic, social,

religious, and political developments with its Western European neighbors. At the same time, the

unique cultural composition of Belgium raised special problems not faced elsewhere in the West.

Page 32: AP Euro DBQ Database

Document 1

Document 2

"The Flemish people of Belgium, inhabiting the country north of a line drawn near the city of

Brussels, are of Teutonic origin. Their ancestors battled against the march of the constantly

invading sea, against the advance of Roman legionaries and the attacks of savage barbarians,

and resisted and defeated them. The savage German barbarians were either absorbed or driven

back; the Roman legions marched over the land but never conquered it, and Roman law, custom,

or language obtained no foothold in the country of the [Dutch-speaking] Flemings.

"To the south of the line lives another race of Celtic origin, the [French-speaking] Walloons.

Supporting the same monarchy, ruled by the same laws, but differing in language, customs,

blood, and traditions, the nearly two million Walloons who live south of the line look upon the

majority Flemings who live north of it as aliens and foreigners. Except upon the arbitrary

divisional line, there is no racial fusion.

"The Celtic Walloon is impatient of political or priestly control, has little regard for political or

religious tradition, but is readily disposed to absorb new faiths and modern politics."

Henry Hilliard, American diplomat in Brussels, 1842

Page 33: AP Euro DBQ Database

Document 3

DISTRIBUTION OF THE BELGIAN POPULATION BY MAJOR LANGUAGE AREAS,

1846-1910

(in millions and by percentage)

1846 1880 1910

Population

Percentage

of Total

Population

Population

Percentage

of Total

Population

Population Percentage of Total

Population

French

Speaking

Areas

1.4 33% 1.3 31% 2.4 32%

Dutch

Speaking

Areas

2.2 51% 3.0 54% 4.1 55%

Brussels

Area

(Bilingual)

.7 16% .8 15% 1.0 13%

Total

Belgian

Population

4.3 100% 5.6 100% 7.5 100%

Extracted from statistical data compiled by the Belgian Ministry of the Interior

Document 4

"In the (Flemish) north, the most ardent supporters of the national government

leadership are the French-speaking upper bourgeoisie; these northern Francophones

[French speakers] are but 5 percent of the region's population, but are an elite that

controls much of Flanders' wealth. Their economic power, especially in commercial

enterprises, is resented by most of the Flemings, who see them as a foreign occupying

power receiving their orders from the Francophones of Brussels."

L'independance belge, a Brussels-base "middle-of-the-road" daily newspaper, 1872

Document 5

"To acknowledge the linguistic rights of the Flemings is to accept the inevitable

consequences of bilingualism. No longer can those who use the French language make

French the predominant language. The improvement of educational facilities for all

Belgians requires real equality in the respect shown the Flemish culture."

Flemish pamphlet, 1879

Page 34: AP Euro DBQ Database

Document 6

"The Flemish are more Roman Catholic and more royalist than those in the south.

Walloons are by nature anticlerical (one might say even bent on the dechristianization of

Belgium) and display only a lukewarm affection for monarchy. The quarrels between the

two in the political arena do not, however, reflect these crucial differences and their

importance, but instead dwell on this language or that language, etc."

French diplomatic observer, 1890

Document 7

"The Flemish have always suffered under a system of economic exploitation, from the

early days of the potato famine in the 1840's and the government's decision to give aid

and relief to paupers. This pattern of 'hand-outs,' together with the absence of national

investments that would improve the entire Flemish region, continues to this day."

C. Smeenk, Flemish political leader in the Chamber of Deputies, 1896

Page 35: AP Euro DBQ Database

Document 8

"The Flemish struggle has been transformed into a struggle for political power, and it is

led by the dominant Catholic party. With the 1893 (universal suffrage) revision of the

Constitution, the Catholics have appealed to a newly enfranchised peasantry of Flanders

which has discovered a connection between the economic decline of their region and the

prosperity of the Walloon provinces The Belgian Workers' party (Socialist political

party), emerging as a weapon to free workers from distress and oppression, has gained

some new supporters with its call for social democracy but has not committed itself to

language reform. The Catholic party, not the Socialist, has understood that the Flemish

movement that aroused the masses was politically so important that its demands had to

be met."

F. Payen, French observer, 1899

Document 9

"For over 50 years, our nation has worked its way, at a rapid pace, toward an industrial

shape. Two sectors of the Belgian economy have evolved: one advanced and comparable

to its major [Western] European neighbors, one less advanced in the spread of steam-

driven machinery. These sectors coincide with the language communities. The Flemish

regions have remained agricultural and commercial; in Wallonia, activity in textiles and

metallurgy is dominant."

Le Moniteur beige, a weekly government publication, 1900

Document 10

"One cannot shrug off the Flemish movement or dismiss it as the eccentricity of a few

extremists. It is not simply a question of who should speak Dutch or French and when

and where. The differences in the two regions come out of different views of history,

religion, and politics, and can be understood only if the economic and social structures of

each area are seen as decisive. If viewed properly, then, the Fleming yearns for real, not

professed, equality of his language and cultural ways."

E. Vandervelde, Belgian political leader, 1902

Document 11

"The conflict between these two language groups-the Dutch-speaking, largely peasant

population of the northwest and the French-speaking, more industrial population of the

southeast-is no longer confined to a small intellectual circle of Flemings and the French-

speaking [central] government in Brussels. The purely scholarly cause is now a political

battle pressing for reforms. It is an assault on French, the language of the rich ruling

classes and the language of atheism. That French is used exclusively by the government,

in the courts, army, universities and secondary schools is a state of affairs we bitterly

resent. We demand nothing less than equal language rights."

Flemish publicist, 1912

Page 36: AP Euro DBQ Database

Document 12

"One thing still separates Fleming from Walloon-the difference of language. Between

the Teutonic Flemings and the German frontier is the French or Walloon speech of

Liege, Namur, and Luxemburg. Roughly speaking, France is held to be a Liberal and

anti-Catholic language; Flemish a Conservative and Catholic one. So there is a great deal

of politics mixed up with the only cause for jealousy which divides the two races.

Flemish has given to Belgium poets like Ledeganck, and has suffered from the defection

to French of Maeterlinck, a Walloon playwright. The Walloons have produced many

savants and historians and among them imaginative writers. There have been French

movements and Flemish movements; and of late years it is the Flemish movement which

has been the more vigorously conducted with effects on education and on politics which

time will show to be important."

The Times of London, May 14, 1913

Page 37: AP Euro DBQ Database

1984 DBQ

Directions: The following question is based on the accompanying Documents 1-13. (Some of

the documents have been edited for the purpose of this exercise.) Write your answer on the lined

pages of the pink essay booklet.

This question is designed to test your ability to work with historical documents. As you analyze

each document, take into account its source and the point of view of the author. Write an essay

on the following topic that integrates the analysis of the documents. You may refer to historical

facts and developments not mentioned in the documents.

The Question: Explain the role of the German army in the development of the German aircraft

industry between 1908 and 1918. Did the army support or hinder the technological development

and the production of aircraft?

Historical background: The first German aircraft firms were founded in 1908. The firms, which

were small in scale, experimented with various designs. In the course of the First World War the

airplane became an indispensable weapon of warfare. By the end of the war, most of the

European powers had recognized the military potential of airplanes.

Page 38: AP Euro DBQ Database

DOCUMENT 1

Excerpt taken from the German army publication, Annual Report on Aviation, September 1908

The German army presently believes that its own work in the field of aviation technology is not

yet absolutely necessary, since no type of flying machine has yet demonstrated its suit-ability for

military purposes. The solution to the problem should therefore be left to private firms with

whom the army should maintain constant contact.

DOCUMENT 2

Taken from a memorandum issued by the German Army Transportation Research Unit, March

15, 1910

The development of the airplane has been the fastest of any modern technical creation. A special

military aviation organization is necessary to make the airplane useful for military purposes.

Otherwise the creation of exceptional aircraft with high performance under good conditions will

occur, but not the creation of aircraft that will be usable and safe in the field under difficult

conditions.

DOCUMENT 3

Taken from a memorandum of the chief of the German general staff to the War Ministry,

November 8, 1911

I am pleased that the newly created military aviation commission provides for prizes, subsidies,

and contests to promote the further development of our domestic aircraft industry. In light of the

anticipated heavy loss of aircraft in the field, the presence of an industry capable of delivering

sufficient and usable replacements in time of war is a vital matter.... I also request that the

military aviation research unit be as well equipped as possible, so that it can test all aircraft

improvements that appear to be militarily worthwhile.

Page 39: AP Euro DBQ Database

DOCUMENT 4

Figures taken from Statistics on Aircraft Manufacture, 1914-1918, German Archives, Freiburg,

1919

DOCUMENT 5

Letter from the War Ministry to the German Aircraft Manufacturers' Trade Association, May 6,

1912, published in the German Aviators' Journal, May 19, 1912

It must be concluded that the army will be almost the sole customer for aircraft in the immediate

future. Therefore, in the opinion of the army, it seems in the interest of the national aircraft

industry that any new factories be large-scale, well-capitalized, and build only types of aircraft

that are assured of success. The army recommends that the Aircraft Manufacturers' Trade

Association impress these conditions upon its members.

Page 40: AP Euro DBQ Database

DOCUMENT 6

Recommendations of the Military Aviation Crash Commission accepted and enacted by the

German army, October 1913

In order to prevent further accidents like Lieutenant Eckenbrecher's recent crash, the aviation

commission proposes that the army proceed in the following manner in the future:

*Subject all army aircraft types to practical weight tests to ascertain their strength.

*Establish and publish requirements (standards) for materials used in aircraft construction.

*Continue to award monetary prizes for instruments that measure the stresses on aircraft in

flight.

In stating the above points, the commission believes that the army flight safety codes will still

allow the factories sufficient freedom to innovate; thus the army will not be saddled with

responsibility for the performance of the manufacturers‟ aircraft.

DOCUMENT 7

Letter from Director W. Froebus of the LFG Aircraft Company to August Euler, aircraft

manufacturer, December 2, 1914

The army recently set new performance requirements for airplanes. Aircraft firms however are

pointing out that they cannot meet these demands overnight and that several months will be

necessary for construction and testing. Furthermore the firms have already prepared the materials

for a planned large series of aircraft machines. These materials must first be used up before the

firms can consider starting a new type of aircraft.

DOCUMENT 8

Memorandum issued by the chief of field aviation of the German army, June 5, 1915

Military aviation must presently rely upon a few factories whose capabilities are totally

inadequate to meet the great demands of the near future. Thus the firms avoid costly

experiments. Progress is retarded, and achievement stands still. Without fail, we must induce our

large industrial enterprises to undertake aircraft construction, either by building new plants or by

buying the most proven small factories and transforming them into large plants.

Page 41: AP Euro DBQ Database

DOCUMENT 9

Anthony Fokker, quoted in Flying Dutchman: The Life of Anthon y Fokker, 1931

In late 1915 I was asked by the army to cooperate with Hugo Junkers, father of the thick wing

and all-metal airplane, in developing an all-metal single-seater monoplane. Professor Junkers is

one of the pioneers in airplane construction. His theories have frequently been in advance of his

time, but he has trouble modifying them for immediate practical purposes. For example, I

persuaded him to construct many parts of his planes of steel tubing, and urged him to permit the

rudder, elevator, and ailerons [movable parts of an airplane wing] to be fabric-covered to

facilitate manufacture. Yet Junkers refused to give up his all-metal construction for the sake of

wartime necessity.

DOCUMENT 10

Anthony Fokker, quoted in Flying Dutchman: The Life of Anthony Fokker, 1931

In the fall of 1916, I concentrated on designing an altogether new and advanced pursuit plane.

Soon I had developed a biplane of radical appearance, with cantilever wing construction which

required no external bracings. It had both speed and climb, and went through all combat

maneuvers like greased lightning. I telephoned a request to army officials in Berlin to come at

once. Staring coldly at my biplane, they walked around it as if it would bite. Someone wondered

idiotically what was going to keep the wings on. They wanted something visible supporting the

wings. This was the sort of thing to which they were accustomed. Desperately I flew the plane as

it had never been flown before. They seemed a little disappointed that the wings hadn't fallen off

in the air to confirm their views. Even the so-called scientific members of the group could not

bring themselves to recommend it for military use. After an entire day of futile argument, I was

forced to admit that the ship was simply too far in advance of its time.

DOCUMENT 11

From a position paper prepared by the German Aircraft Manufacturers' Trade Association in

defense of patent protection during wartime, May 30, 1916

With regard to aviation technology and research, more was achieved before the war for the war

than has been achieved during the war for the war. Those changes that the war has brought about

are limited more to specialized technical matters, such as bombing, photography,

radiotelegraphy, instrumentation, and accessories. The enlargement and development of the

aircraft industry caused by the war contracts is almost completely a matter of expanded

production.

Page 42: AP Euro DBQ Database

DOCUMENT 12

From a memorandum on German and Allied aviation from the records of the United States

Army Air Force, September 20, 1918

The German army has controlled its aircraft manufacturers by a central technical organization

which has given the general contractors technical instructions to follow. These directions have

permitted a reduction in the number of types of military aircraft being produced. The results of

this method are perhaps a slower improvement than ours in machines, but a steadier and more

equal advance by the different manufacturers. Builders have been brought in contact one with the

other, and they cannot keep their discoveries or the result of their studies to themselves. In other

words, the Germans have brought about a far greater standardization in construction than have

the Allies. Standardization has simplified and cheapened production.

DOCUMENT 13

Richard Blunck, Hugo Junkers: A Life for technology and Air Travel, 1951

It was often asserted that Hugo Junkers‟ firm had made huge profits during the war. One can

only reply that it was not a matter of profits as huge as other armament firms, but that Junkers

ploughed his profits completely into new research work. Junkers maintained the essence of his

enterprises-research. Research tasks dominated, enabling his firm to recover quickly after the

apparently hopeless collapse of aircraft factories at the end of the war.

Page 43: AP Euro DBQ Database

1985 DBQ

Directions: The following question is based on the accompanying Documents 1-13. (Some of the documents have been edited for the purpose of this exercise.) Write your answer on the lined pages of the pink essay booklet. This question is designed to test your ability to work with historical documents. As you analyze each document, take into account its source and the point of view of the author. Write an essay on the following topic that integrates the analysis of the documents. You may refer to historical facts and developments not mentioned in the documents.

The Question: To what extent did changing views on the causes of juvenile crime affect legal treatment of the juvenile offender in nineteenth-century Great Britain?

Historical background: Early in the nineteenth century, the British government, re-sponding to the public‟s fear that lawlessness was increasing, took measures to reduce crime. For example. it established a professional police force and created committees to study the justice system. Generally, punishments were based on the assumption that crime stemmed from an individual‟s weak or evil character.

LEGAL TREATMENT IN THE EARLY NINETEENTH CENTURY

Document 1

On the day the members of the Committee visited Tothill Fields Prison, they found that those who had committed misdemeanors and assaults were confined together in a yard which opened into one where the vagrants were detained. In that yard were two children, one 11 and the other 12. In the same yard were two persons imprisoned for assault with intent to perpetrate an unnatural offense.

Upon a review of the present state of Clerkenwell Prison, your Committee finds that out of the 4,063 prisoners who were sent there during the last year, 470 were under 20 years of age, charged with felony. And of these, one was as young as 8, and sixty-nine were under 14 years of age.

Select Committee Report on the Prisons of

London,

Parliamentary Papers, 1818

Page 44: AP Euro DBQ Database

Document 2

T.G.B. Estcourt, Esquire, Chair of the Select Committee on the Police of the Metropolis:

Do you not think there is a great evil in committing a young boy to Newgate Prison for simple larceny, and leaving him there two months before he is brought to trial?

Sir Richard Birnie, Chief Magistrate in Middlesex County:

Yes, very great, to Newgate or any other prison.

Estcourt:

In what way would you advise that young boys should be punished? Birnie:

I should recommend a little flogging at a certain age. Estcourt:

You think transportation to Australia is not a fit punishment for a boy of fourteen or fifteen years of age?

Birnie:

I am talking of transportation as inappropriate for those of nine and ten years of age.

Minutes of evidence before the Select Committee on the Police of the Metropolis, Parliamentary Papers, 1828

Document 3

At the present time, there are three children, one 7, one 8, and another 10, all of whom were undergoing confinement in separate cells in the Clerkenwell penitentiary. The youngest child has been convicted at Manchester of having stolen certain goods which the mother had received, knowing them to be stolen property. The child was sentenced to transportation and the mother to six months‟ imprisonment. Since the child‟s years would furnish sufficient proof that she could not be fully aware of the nature of her offense, the child‟s sentence was commuted to solitary imprisonment under the separation system.

Testimony of Mr. Hoare, a visiting justice at the Clerkenwell House of Correction, to the London Court Sessions, 1838

Page 45: AP Euro DBQ Database
Page 46: AP Euro DBQ Database

Document 5

Delinquents are born thieves. It is their inheritance. They form a caste of themselves, having

their peculiar slang, mode of thinking, habits, and art of living.

John Wade, A Treatise on the Police and Crimes of the Metropolis, London, 1829

Document 6

Young thieves have often confessed to me that their first attempts at stealing began at apple stalls. Acquiring confidence by a few successful adventures, they have gradually progressed in crime. They find companions to cheer them and instruct them, girls to share their booty and applaud them. Imprisonment is no punishment. It‟s no matter to him where he exists as long as he has food and some clothing. In fact, many lads have admitted to me that they learned more in jail than out of it.

Mr. W.A. Miles, Esquire, Report on Prison Discipline, presented to the House of Lords, 1835

Document 7

One grand cause of depravity and crime in children is the vice of their parents, who often educate their offspring in the art of thieving and live upon the proceeds of their children‟s depredations. In speaking of three children whose lives of crime he relates, Mr. Rushton observes, “These lads have been trained by a vicious father to the work of plunder. He has taught them how to steal with dexterity, and he uses them as a means of supplying himself with a luxurious existence.” Unless the evil power of the parent be destroyed and his mischievous teaching counteracted, it is clear that no valid hopes of reformation can be obtained.

Report of an address to the Town Council of Liverpool by its magistrate, Mr. Rushton, The Times of London, August 21, 1850

Page 47: AP Euro DBQ Database

Document 8

The scanty wages given to many forms of labor, as well as the high price of rent and provisions, make it almost impossible for a man alone to support the family. Hence, most of the wives of the unskilled workpeople have to forego their maternal duties, and devote themselves to some kind of drudgery to add to the petty household income. If then the mother be away from home the greater part of her time, and the children be left to gambol in the gutter with others as neglected, what reward can society look for from this moral anarchy and destitution? Here is the real explanation of juvenile delinquency.

H. Mayhew and J. Binny, The Crimina Prisons of London, London, 1862

LEGAL TREATMENT IN THE SECOND HALF OF THE NINETEENTH CENTURY

Document 9

When any person under the age of sixteen shall be convicted of any offense, it shall be within the power of any Court, in addition to the sentence passed as a punishment for his offense, to direct such an offender to be sent at the expiration of his sentence to one of the Reformatory Schools, for not less than two years and not exceeding five years.

Youthful Offenders Act, 1854

Document 10

If you want success, follow the new plan of family division. Separate and distribute the inmates into households. Place their dwellings at a considerable distance from each other. Give them separate fields of labor. Make each family a complete school or institution on a small scale by itself.

There is nothing like gardening and farm-work for giving a new direction to the young criminal‟s tastes and habits. Handling a spade spoils the fingers for the delicate operations of pickpocketing. And the sights and sounds of Nature turn interest from these activities.

Letter of the Reverend Sydney Turner, in charge of the Redhill Reformatory School, to Mr. C.B. Adderly, Member of Parliament, circa 1855

Page 48: AP Euro DBQ Database

Document 11

There can be no doubt that the Reformatory Schools, as a result of the Act of 1854, are doing important work in arresting crime in the country. They also confer a great benefit on the young persons who seem destined without help to a life of crime. But the entrance to the Reformatory is through the prison first. It therefore appeared increasingly important to establish the Industrial School where young persons could be sent by a Magistrate without some time spent in jail. Such a measure was obtained on August 17, 1857, when the Industrial Schools‟ Act* was passed.

Mary Carpenter, Our Convicts, 1864

*This legislation provided for the education and vocational training of neglected or delinquent children, under 14 and generally under 12 years of age, when they were committed by a court. A child could be committed to an industrial school for such a period as the court thought proper, but not beyond the age of 16.

Document 12

Industrial Schools are credited, we believe justly, with having broken up the gangs of young criminals in the larger towns; with putting an end to the training of boys as professional thieves; and with rescuing children fallen into crime from becoming habitual or hardened offenders. Undoubtedly, they have also had the effect of preventing a large number of children from entering a career of crime.

Royal Commission Report,

Parliamentary Papers, 1884

Document 13

The proportion of children now sent to prison is much smaller than it was. Besides this, the sentences passed on children are much lighter than they were, and a check is exercised on the Magistrates. As an instance, I will quote the case of a boy sentenced to a month‟s imprisonment for stealing fruit. His term of imprisonment was changed from a month to seven days by the Home Office which reviews Magistrates‟ sentences. The practice of flogging nevertheless still exists, and not long ago a policeman told me, with relish, that he had birched (whipped) as many as sixty boys on a single day at one court session. Still, there is a growing tendency to make grave distinctions between the treatment of juvenile and adult criminals. An outcome of this tendency may be found in the creation of Reformatories and Industrial Schools for younger and, as a rule, more innocent children. A system of rewards in money obtains both in Reformatory and Industrial Schools, so that by good conduct a child may accumulate a small sum for the time when its period of detention shall be over. The necessity for corporal punishment is minimized by the system of rewards.

Gertrude M. Tuckwell, The State and Its Children, London

1894

Page 49: AP Euro DBQ Database

1986 DBQ

Directions: The following question is based on the accompanying Documents 1-13. (Some of the

documents have been edited for the purpose of this exercise.) Write your answer on the lined

pages of the pink essay booklet.

This question is designed to test your ability to work with historical documents. As you analyze

each document take into account its source and the point of view of the author. Write an essay on

the following topic that integrates the analysis of the documents. You may refer to historical

facts and developments not mentioned in the documents.

1. Analyze the pressures on Great Britain's Liberal government during the Sudan crisis (1884-

1885), and explain why the government acted as it did.

Historical background: In 1883 a major crisis arose in the Sudan, the territory south of Egypt

under Egyptian control. A Muslim religious leader known as the Mahdi led a popular rebellion

against Egyptian rule and foreign influence in the Sudan. The rebellion attracted the attention of

the British government, which had become deeply involved in Egypt's affairs by 1880. From

1880 to 1885 the Liberal party under the leadership of Prime Minister William Gladstone was in

power in Great Britain.

Page 50: AP Euro DBQ Database

Document 1

EGYPT AND THE SUDAN, 1884

Page 51: AP Euro DBQ Database

Document 2

General Gordon started last night for Egypt. His immediate purpose

is to report on the military situation in the Sudan, to provide in the

best manner for the safety of the European population of the capital

Khartoum and the Egyptian garrisons [permanent military

installations] still in the country, as well as [to provide] for the

withdrawal of all Egyptian military units from the Sudan. To do

what is needed, General Gordon is probably better suited than any

living man. His name is respected throughout Egypt, and his prestige

in the Sudan has been unrivalled since he served as governor there

between 1877 and 1879.

The Times, London, January 19, 1884

Document 3

The Marquis of Salisbury (Conservative party leader in the House of

Lords debate) February 12, 1884:

The dishonor upon the Egyptian army and upon our English officers

caused by the defeats at the hands of the Madhi comes from

inconsistency in the British government's policies. The Gladstone

government began with a sound and sober policy of recognizing the

Sudan as Egypt's possession. It went on to a policy of indifference

and fear of responsibility for the Sudan, and it is ending with a

policy of panic. You, the members of Her Majesty's government, do

not care for empire; nothing is so pleasing to you as evacuation. How

much this will contribute to British influence in the Near East and to

the stability of your rule in India, I leave to your lordships to

imagine.

Document 4

Sir George Campbell (Liberal party member of Parliament, from the

debate in the House of Commons) February 12, 1884:

I earnestly hope that the government will not enter on a great war in

the Sudan, but will rest content with rescuing the garrisons. I protest

against going to war with the Madhi; I protest against a war of

Christianity against Islam in the Sudan. We should get out of Egypt

as soon as we possibly can, and not be deterred by the concerns of

the European bondholders and the creditors.

Page 52: AP Euro DBQ Database

Document 5

Sir Wilfred Lawson (Liberal party member of Parliament, from the

debate in the House of Commons) February 14,1884:

Who got us into this trouble in Egypt and the Sudan? The

Conservative party's leader began it in 1882, when he went to a

meeting presided over by the chairman of the British holders of

Egyptian government bonds and made fire and thunder speeches.

The present Liberal government has been trying to get out of Egypt,

and they cannot get out. The Egyptians do not want our diplomats,

our bondholders, and our European residents governing them. They

want us to go away.

Document 6

The position of General Gordon, besieged at Khartoum, unfortunately remains

exceedingly precarious. Yesterday we published the unwelcome report that he is now

totally isolated. All communications have been cut. A month ago the British government

was fully warned that it would become necessary to employ something more than moral

force at Khartoum. The necessity is now becoming urgent, but the government has not

yet ordered the commander of British and Egyptian forces in southern Egypt to march to

Khartoum.

Commentary in The Times, London, March 20, 1884

Document 7

In the spring of 1884,the Liberal party was divided on the question

of the Sudan, some wishing for an expedition to rescue Gordon at

Khartoum, others hot for abandonment of him. The Conservatives

were divided, too; most of them probably wished for an expedition,

but they were afraid to say so.

Memoirs of Sir Charles Dilke, member of the Gladstone cabinet,

published in 1917

Page 53: AP Euro DBQ Database

Document 8

10 Downing Street; 23 April 1884

We ought to act in the Sudan only by peaceful means, except for the

safety of Gordon and his party. If, in consequence of his being in

danger, we have to act by military means, the object of our action

ought to be to bring him away at once from Khartoum, and he ought

to know that.

If Gordon continues at Khartoum knowing that we cannot approve of

supplying him with any forces for military expeditions, he should

state to us the cause of his staying and his intentions.

Memorandum from Prime Minister Gladstone to Foreign Secretary

Lord Granville

Document 9

Sir Michael Hicks-Beach (Conservative party member of

Parliament, from the debate in the House of Commons) May 12,

1884:

I believe that the people of this country are determined that General

Gordon shall be saved together with those who have trusted in him.

(Cheers) If General Gordon had been supplied with materials of war

earlier, he would have been enabled to stem at Khartoum the wave of

religious fanaticism and anarchy led by the Madhi.

It is our duty to complete the commitments which we made when

General Gordon went out to the Sudan. Her Majesty's Government

must leave no stone unturned to avert from this country the

intolerable stain which would be left upon her honour by any injury

inflicted upon General Gordon.

Page 54: AP Euro DBQ Database

Document 10

Prime Minister Gladstone (from the debate in the House of

Commons) May 12, 1884

The government was and is pledged to shield General Gordon from

danger. Should necessity arise, the government shall do this. The

Right Honorable Gentleman Sir Michael Hicks-Beach has said,

though, that it is the duty of England to keep the Mahdi's movement

out of Egypt and to put it down in the Sudan, and it is this task which

the gentleman desires to saddle upon England. That means the

conquest of the Sudan. I put aside for the moment all questions of

climate, of distance, of the enormous expenses, and all the frightful

loss of life. There is something worse involved. It would be a war of

conquest against a people struggling to be free.

Document 11

It was not until August 8, 1884, that the British secretary for war

authorized preliminary steps for moving troops south from Egypt

into the Sudan. While authorizing these actions, however, the

government stated in Parliament: "Her Majesty's government is not

at present convinced that it will be impossible for General Gordon to

withdraw from Khartoum." On August 26, Lord Wolseley was

appointed to command the expedition to rescue Gordon.

Sir Evelyn Baring, British Consul General in Egypt, 1883-1907,

Modern Egypt, I, (1908)

Page 55: AP Euro DBQ Database

Document 12

"Too Late!"

Telegram, Thursday Morning, Feb 5. "Khartoum taken by the

MAHDI. General GORDON'S fate uncertain"

Page 56: AP Euro DBQ Database

Document 13

I have had the honour to receive Your Majesty's telegram, stating

that it is too frightening to consider that earlier action might have

prevented the fall of Khartoum and saved many lives. I am not

altogether able to follow Your Majesty's conclusion. Your Majesty's

ministers were well aware that climate and distance were far more

formidable than the sword of the enemy. Probably abundant anger

will be poured out on Your Majesty's ministers, but a partial

consolation may be found in the fact that no gross error in the

application of means to ends has marked these difficult proceedings.

Excerpt of a letter from Prime Minister Gladstone to Queen

Victoria, February 5, 1885

Page 57: AP Euro DBQ Database

1987 DBQ

DIRECTIONS: The following question is based on the accompanying Documents 1-14. (Some

of the documents have been edited for the purpose of this exercise.) Write your answer on the

lined pages of the pink essay booklet.

This question is designed to test your ability to work with historical documents. As you analyze

each document, take into account its source and the point of view of the author. Write an essay

on the following topic that integrates your analysis of the documents. You may refer to historical

facts and developments not mentioned in the documents.

THE QUESTION: Describe the variations in the levels of literacy in Old Regime France and

trace these variations over time. Analyze the factors that promoted or discouraged the spread of

literacy.

Page 58: AP Euro DBQ Database

Document 1

LITERACY IN FRANCE

Adapted from Francois Furet and Jacques Ozouf, Reading and Writing: Literacy in France from

Calvin to Jules Ferry, 1982."

Page 59: AP Euro DBQ Database

Document 2

PERCENTAGE OF FRENCH VILLAGES WITH SCHOOLS, BY SELECTED REGIONS

(1730-1789)

From pastoral visits, bishops' inquiries and reports by intendants, 1730-1789

Document 3

LITERACY IN FRANCE BY SOCIAL GROUP

1680's 1789

Male Female Male Female

Nobels and Professionals 95% 60% 95% 85%

Artisans 50% 20% 80% 50%

Prosperous Peasants 20% 0% 45% 10%

Sharecroppers and

Laborers 5% 0% 15%` 0%

Page 60: AP Euro DBQ Database

Document 4

I found scattered farmhouses in Northern France. All are collected in villages: small towns and

villages are everywhere in sight. This region contains the cream of French agriculture: the soil is

excellent. The vast range of country is an unbounded plain, the whole scattered with rich

meadows, vineyards, gardens and forests. Ready markets for every sort of produce are an

incentive for improvement.

In contrast, in the south of France, the country is all hill or valley, farmhouses are everywhere

scattered instead of being collected in towns and farming here is backward. The poor people who

cultivate the soil here are sharecroppers, a miserable system that perpetuates poverty and

excludes instruction. This unimproved, poor and ugly region of the country seems to lack

communication, demand and activity of all kinds: nor does it yield on the average the half of

what it might.

Arthur Young, English agricultural expert, Travels in France 1770's - 1780's

Document 5

Although their souls are equally precious to God, an aristocratic young lady should receive

broader instruction than a wine grower's daughter: a wine grower's daughter need only know

what is absolutely essential for salvation, but aristocratic young ladies need further instruction.

Wine growers' daughters will make themselves ridiculous by reciting verses, while poetry is

good for aristocratic young ladies.

* * * *

Merchants' daughters should learn about exactitude in commerce, measures weights and the

allowed profit. Neither poetry nor conversation is necessary for the daughters of the bourgeoisie;

it is not a question of embellishing their minds.

Francoise d'Aubigne, Marquise de Maintenon

Founder of a school for aristocratic women, Excerpts of letters, 1715

Page 61: AP Euro DBQ Database

Document 6

The school procures immense benefits, spiritual as well as temporal, by instructing youth,

because by this means a great number of children learn writing and arithmetic, which equips

them to engage in commerce, which is the only resource they have for pulling themselves out of

poverty.

A French Bishop, Letter, 1737

Document 7

The unremitting and arduous physical labors to which peasants are destined for cultivation of

the land and for hauling do not at all demand that they know how to read, and even less that they

know how to write. As long as a peasant can form the letters of his name, he is skilled enough;

and what he might know beyond that is of no utility whatever to him.

A government official in southern France, Letter, 1738

Document 8

I know that one argues in favor of schools on the pretext of religion: saying that it is important

that peasants know and understand on their own what they should believe. But shouldn't the

guidance of their priests suffice for that? In all centuries and especially in ours, knowledge has

made few good believers and has led many others astray.

Excerpt from a government report, 1744

Document 9

Schools should not be open except on holidays and on Sundays after church during the summer

and at night during the winter. Most people would then know nothing: hooray for that! They

would know neither how to read nor how to write, but they would have arms accustomed to

digging the earth: that would make workers and soldiers.

Francois Quesney, a leading French Physiocrat

Essay on Land Management 1759

Page 62: AP Euro DBQ Database

Document 10

Today, even the lower classes want to study. Laborers and artisans send their children to

schools, and when they have received a wretched education, which has taught them merely to

despise their fathers' trades, they fling themselves into the monasteries, become priests or

officers of justice, and frequently turn out to be a danger to society. Schoolmasters teach reading

and writing to people who should never have learned more than a little drawing or how to handle

the plane or the file, and who now don't want to do this.... The good of society requires that the

knowledge of the people not extend farther than its occupations. A man who sees beyond his

own sad profession will never ply it with courage and with patience. Among the people, it is

scarcely necessary to know how to read and write.

A French aristocrat and jurist, Essay on National Education or Plan of Studies for Youth,

Presented to a provincial parlement, 1763

Document 11

It is not possible to form true worshippers of God and faithful subjects of the king without the

help of instruction. Nor is it possible for priests to instruct in the faith those coarse inhabitants of

the countryside who do not know how to read. We have the sorrow of seeing that these young

people who do not know how to read forget soon after their first communion even the most

elementary parts of religion that they had learned in their childhood.

Priests writing to their archbishop, 1769

Document 12

All women should be prohibited from learning to write and even read. This would preserve them

from loose thoughts, confining them to useful tasks about the house, instilling in them respect for

the first sex.

Restif de la Bretonne, A French author, 1777

Page 63: AP Euro DBQ Database

Document 13

I have always found that it was better if there were no schools at all in the villages. A peasant

who knows how to read and write leaves agriculture, which is a great evil.

An intendant, Letter 1782

Document 14

A great part of the ills which we have suffered would not have existed or would not have been

so lasting if the inhabitants of the countryside had been better instructed, had been able to read

regularly the good or bad laws and to express their own clear observations on the good and evil

which would result.... It would thus have been a great public benefit, a great service to the

government, a great remedy against a crowd of abuses to teach the inhabitants of the countryside

to read, to write, and to calculate with facility. The long winter evenings would make many

peasants hardworking and studious. New agricultural knowledge could spread on from province

to province. The proprietors and inhabitants of this parish think that it would be very useful if

there were in each village a good schoolmaster capable of teaching children reading, writing,

arithmetic, surveying, and measuring.

Cahier (a list of grievances) of a village near Paris, 1789

Page 64: AP Euro DBQ Database

1988 DBQ

Directions: The following question is based on the accompanying Documents 1-13. (Some of

the documents have been edited for the purpose of this exercise). Write your answers on the

lined pages of the pink essay booklet.

This question is designed to test your ability to work with historical documents. As you analyze

each document, take into account its source and the point of view of the author. Write an essay

on the following topic that integrates your analysis of the documents. You may refer to historical

facts and developments not mentioned in the documents.

1. Analyze the arguments for and against the restriction of the sale of gin in eighteenth century

England, and assess the degree to which the issues of the debate were reflected in the following

excerpt from the Gin Act of 1751.

"Whereas the immoderate drinking of distilled liquors by persons of the meanest and lowest sort,

hath of late years increase, to the great detriment of the health and morals of the common people,

the commons of Great Britain in Parliament assembled, ever attentive to the preservation and

health of your Majesty's subjects, have taken this great evil into our serious consideration, and

proposed such laws and provisions as appear to us to be more likely to put a stop to the same."

From the Preamble of the Gin Act of 1751

Historical Background: After the Glorious Revolution in 1688, gin began to rival beer as the

most popular drink in England. In 1689 the English government opened the distilling trade to all

English people who paid certain taxes. Over the next sixty years, however, the government

regulated the sale of gin with an inconsistent taxation policy. With the Gin Act of 1736, for

example, the government imposed a high license fee for gin retailers and a high per-gallon tax;

yet, the license fee and the tax were significantly lowered within a few years. Finally the Gin Act

of 1751 (a) prohibited gin distillers from selling to unlicensed merchants, (b) restricted retail

licenses to substantial property holders, and (c) charged high fees to those merchants eligible for

retail licenses.

Page 65: AP Euro DBQ Database

Document 1

Customs Library, Excise Revenue Accounts, 1701-1751

Document 2

Nothing is more certain than the fact that the ordinary production of grain in England is

much greater than our people or cattle can consume. Because gin is made from grain, the

distilling trade is one remedy for this disaster as it helps to carry off the great quantity of

grain in such a time of plenty. In times of plenty and a moderate price of grain, the

distilling of grain is one of the most essential things to support the landed interest and

therefore especially to be preserved.

Daniel Defoe, author and social commentator, Review, 1713

Document 3

Everyone who now passes through the streets of the great city of London and looks into

the gin shops must see, even in shops of creditable and wholesome appearance, a crowd

of poor ragged people, cursing and quarreling with one another over repeated glasses of

these destructive liquors. In one place not far from East Smithfield, a trader has a large

empty room where, as his wretched guests get intoxicated, they are laid together in

heaps, men, women and children, until they recover their senses, when they proceed to

drink on, or having spent all they had, go out o find the means to return to the same

dreadful pursuit.

Anonymous Distilled Liquors: the Bane of the Nation, 1736

Page 66: AP Euro DBQ Database

Document 4

Let us consider, Sir, that the gin trade has been carried on for about 100 years and that it

has been very much encouraged by several acts of Parliament. No one could imagine that

the trade is in itself detrimental to the health and welfare of the people. Accordingly,

great numbers of his Majesty's subjects, especially within the last 40 years, have entered

this business. There is not an inn, an alehouse, or a coffeehouse in the kingdom, but what

owes a great part of its profits to the sale of gin. there are now multitudes of families in

the kingdom who owe their chief if not their only support tot he distilling, or to the sale

of such liquor. They deserve the care and the consideration of the British House of

Common. I cannot give my consent to any regulation which will put them out of the

business to which they owe their chief support.

William Pulteney, landowner, speech in Parliament, 1736

Document 5

This year's Gin Act with its very high license fees strikes at the very root of property

rights and is a prelude to a general excise tax at the next session of Parliament.

Letter to John Moore, an important distiller, 1736

Document 6

His Majesty's share of the taxes upon gin has amounted to 70,000 pounds a year for the

last eight years. Because the proposed Gin Act of 1736 will raise fees so high that

nobody will pay them, His Majesty will lose a yearly revenue of 70,000 pounds. This

probable loss ought to be made known to His Majesty.

Speech by a member of Parliament, 1736

Document 7

We have observed:

That the drinking of gin, and other distilled liquors has for some years greatly increased;

That the constant and excessive use thereof has destroyed thousands of his Majesty's

subjects;

That great numbers of others have been by its use rendered unfit for useful labour,

debauched in morals, and drawn into all manner of vice and wickedness.

County Magistrates from Middlesex, Petition to Parliament, 1736

Page 67: AP Euro DBQ Database

Document 8

I never heard that a single drink, even of the pernicious liquor called gin, was either a

crime or a sin. Whatever some physicians may say, others will tell you that a moderate

drink of some sort of liquor is necessary upon many occasions for the relief or support of

nature. the cold, damp and often foggy climate of England makes such an aid to nature

absolutely necessary.

Lord Bathurst, 1737

Document 9

These liquors not only infatuate the mind but poison the body; they not only fill our

streets with madness and our prisons with criminals, but our hospitals with cripples.

Those women who riot in this poisonous debauchery are quickly disabled from bearing

children or else produce children diseased from their birth.

Lord Lonsdale, speech in Parliament, 1743

Document 10

There is only one condition required in those who desire into our religious society: a

desire to flee from the wrath to come, and to be saved from their sins. It is expected of all

who continue therein, that they should avoid evil of every kind: the taking of the name of

God in vain; the profaning the day of the Lord; drunkenness; fighting; quarreling;

brawling.

John Wesley, A Plain Account of the People Called Methodists, 1749

Page 68: AP Euro DBQ Database

Document 11

GIN LANE

"Gin, Cursed Fiend, with Fury Fraught..."

William Hogarth, Gin Lane, 1751

Page 69: AP Euro DBQ Database

Document 12

BEER STREET

"Beer, Happy Produce of Our Isle..."

Willian Hogarth, Beer Street, 1750

Page 70: AP Euro DBQ Database

Document 13

We make ten times the quantity of gin we made forty years ago. The cheapness of gin

encourages the poor to drink. If the evil of gin increases in the next ten years as it has

done in the last, drunkenness must become the characteristic of the people; they must

live upon spirits, and forget labour and sobriety. The children must be born in gin,

brought up in a gin shop, live in drunkenness and kick out of the world without having

enjoyed one sober thought; but private vices are public benefits, and while they continue

such, we have no great hopes of redressing those many calamities that attend national

drunkenness.

Excerpt from The London Tradesman, 1747

Page 71: AP Euro DBQ Database

1989 DBQ

Directions: The following question is based on the accompanying Documents 1-13. (Some of the documents have been edited for the purpose of this exercise.) Write your answer on the lined pages of the pink essay booklet. This question is designed to test your ability to work with historical documents. As you analyze each document, take into account its source and the point of view of the author. Write an essay on the following topic that integrates your analysis of the documents; in no case should documents simply be cited and explained in a “laundry list” fashion. You may refer to historical facts and developments not mentioned in the documents.

The Question: In the nineteenth and twentieth centuries, women in many European countries organized and agitated for greater political, legal, and economic rights. The debate that developed around the many issues raised by organizers for women‟s rights became known as the “woman question.” In particular, people had strong opinions about whether women should vote.

Analyze AND compare the major points of view concerning suffrage and the ways in which individual commentators believed woman suffrage would affect the political and social order.

Page 72: AP Euro DBQ Database

Document 1

The family is a school of despotism. in which the virtues of despotism, but also its vices, are largely nourished. Even with true affection, authority on the one side and subordination on the other prevent perfect confidence. Thorough knowledge of one another hardly ever exists, except between persons who, besides being intimates, are equals. The moral regeneration of mankind will only really commence when the family is placed under the rule of equal justice and when human beings learn to cultivate their strongest sympathy with those who are equal in rights.

John Stuart Mill, The Subjection of Women,, 1869

Document 2

When the people acquired the right to vote in 1848, thinking individuals understood that the exclusion of half of humanity was a gratuitous insult for women, equal to men as French persons, equal before taxes both direct and indirect as well as before the penal code, and that it was ironic to call universal a suffrage that rejected women as minors lacking in judgment.

Julie Daubiê, French political activist, 1870

Document 3

Is it not true that in all matters connected with the army, the navy, and matters commercial, diplomatic, and legal, women would have to make judgments on the basis of information obtained second hand, and not from practical experience? If this is true, and if suffrage is adopted, the influence of the clergy over women will be enormous.

Henry James, speech in the House of Commons, 1871

Document 4

The woman question affirms more profoundly the roots of democracy, discredits the rule of force, advances women in the economic sphere, and weakens the power of traditional prejudices.

Anna Mozzoni, leading Italian feminist, The Question of the Emancipation of Women in Italy, 1871

Page 73: AP Euro DBQ Database

Document 5

We cannot separate domestic politics from social conditions of life. If we are told that we have nothing to do with politics, we can but answer that politics have a great deal to do with us. With respect to the home as woman‟s natural sphere, it is by no means a domain she rules, for as wife and mother she has hardly any legal rights. This distinction seems to result in man‟s keeping the supremacy in every sphere to himself.

Arabella Shore of the London

National Society for Women‟s

Suffrage, 1877

Document 6

When, after a political struggle in parliament, you retreat into your domestic sanctuary, you find peace; the day when women participate in public business, you will find war.

Francesco Crispi, liberal Italian politician and future prime minister, speaking in the Italian Senate, 1883

Document 7

Women will be more valuable citizens, will contribute more precious elements to the national life without the vote than with it. The quickness to feel, the willingness to lay aside prudential considerations in a right cause, which are amongst the peculiar excellencies of women, are in their right place when they are used to influence the more highly trained and developed judgment of men. But if this quickness of feeling could be immediately and directly translated into public action, the risks of politics would be enormously increased.

Mrs. Humphrey Ward, popular novelist, on behalf of a group of prominent English women, “An Appeal Against Female Suffrage,” 1889

Page 74: AP Euro DBQ Database

Document 8

Page 75: AP Euro DBQ Database

Document 9

We hope that the representatives will support the people, give them land and freedom, and open the doors of the prisons to liberate the fighters for the people‟s freedom and the people‟s happiness. We hope that the representatives obtain civil and political rights for themselves and for us Russian women, who are unfairly treated and without rights even within our families. Remember that a slave cannot be the mother of a free citizen.

Petition from the peasant women of Nogatkino to their deputy in the Russian Duma, 1906

Document 10

It is the duty of women in the socialist movement in all countries to participate with all their energy in all battles waged by the socialist parties for the democratization of the suffrage. The obtainment of suffrage helps the bourgeois women to tear down the barriers in the form of male prerogatives which tend to limit women‟s educative and professional opportunities. It arms the female proletarians in their battle against class exploitation and class rule, and in their effort to acquire their full humanity.

We socialists do not only demand women‟s suffrage as a natural right with which women are born; we demand it as a social right.

Clara Zetkin, German socialist leader, “Women‟s Right to Vote,” 1907

Document 11

Women want to rule and we don‟t want to let them. The German Empire was created with blood and iron. That was man‟s work. If women helped, it was not women of the sort involved in the new women‟s movement, but women of the Spartan and old Germanic kind, who stood behind their men in battle and fired them on to kill as many enemies as possible [fervent applause].

Count Reventlow, addressing the German League for the Prevention of the Emancipation of Women, 1912

Page 76: AP Euro DBQ Database

Document 12

Rather than handling the ballot, the hands of women are meant to be kissed----kissed devotedly when women are mothers, and lovingly when wives or fiancees

Speaker for a French Senatorial Commission examining a bill on women‟s suffrage, about 1919

Document 13 This false liberty and unnatural equality with the husband is to the detriment of the woman herself, for if the woman descends from her truly regal throne, to which she has been raised within the walls of the home by means of the Gospel, she will soon be reduced to the old state of slavery and become as among the pagans, the mere instrument of man.

Pope Pius XI, encyclical On Christian Marriage in Our Day, 1930

Page 77: AP Euro DBQ Database

1990 DBQ

Directions: The following question is based on the accompanying Documents 1-16. (Some of the documents have been edited for the purpose of this exercise.) Write your answer on the lined pages of the pink essay booklet. This question is designed to test your ability to work with historical documents. As you analyze the documents, take into account the source and the point of view of the author. Write an essay on the following topic that integrates your analysis of the documents; in no case should documents simply be cited and explained in a “laundry list” fashion. You may refer to historical facts and developments not mentioned in the documents.

The Question: Describe and analyze the ways in which the defenders of the

Spanish Republic represented their aims and their attitudes. Be sure to include in your analysis the differences within and among the groups presented by the documents.

Historical Background: In February 1936 a general election in the Republic of Spain brought to power a government supported by a coalition of center and left-wing parties. A few months later, conservative military officers rose against the government of the Republic, and civil war followed. Over the next three years, conservatives and fascists attempted to overthrow the Republic. Among the groups resisting this attack were moderate republicans, socialists, communists, anarchists, and nationalists seeking autonomy for the Basque region or for Catalonia. The forces supporting the Republic were finally defeated in 1939.

Page 78: AP Euro DBQ Database

Documents 1-4: REPUBLICANS

Document 1

We want no dangerous innovations. We want peace and order. We are moderates.

Manuel Azana. prime minister, Liberal Republican party speech, 1936

Document 2

The Republic attempted to introduce a new modern political mentality through moderate reform.

Unfortunately, the Socialist Left tried to turn from this process.

Andres Marquez, civil servant and member of the

Liberal Republican party, recollection, 1936

Document 3

Workers‟ wives did their grocery shopping without paying for anything for the very good reason that they were accompanied by toughs waving eloquent revolvers. Men and women—republican volunteers—aimed their rifles at passersby and at windows.

Clara Campoamor. centrist republican deputy in the parliament, The Spanish Revolution As Seen by a Republican. 1937

Page 79: AP Euro DBQ Database

Document 4

Page 80: AP Euro DBQ Database

Documents 5-6: SOCIALISTS

Document 5

After the victory of the Liberal Republicans in the General Election of 1936, the government failed to produce a coherent program of social and agrarian reform. In those months, the Socialist party made a big mistake; it should have joined the government. Without renouncing the aim of taking power, the Socialist party—the most Marxist in Europe—could have shared governmental power. The left wing of the Socialist party blocked this. Divided, the Socialist party was not able to channel or lead the revolutionary ferment.

Socrates Gomez. leader of the Socialist Youth Movement, recollection, 1936

Document 6

In fighting for democracy in our country, we are defending democracy in all countries. This is a struggle of two civilizations, of Christianity against Fascism.

Dr. Juan Negrin, prime minister, moderate socialist, speech, February 1, 1939

Documents 7-9: COMMUNISTS

Document 7

We shall follow the path of completing the bourgeois democratic revolution until it brings us to a situation in which the proletariat and the peasantry themselves assume the responsibility of making me people of Spain as happy and free as are the Soviet people, through the victorious achievement of socialism, through the dictatorship of the proletariat.

Worker’s World, Communist party

newspaper, February 1936

Page 81: AP Euro DBQ Database

Document 8

The battle of the Spanish people is the battle of a people that has risen against the criminal aggressions of the reactionary military castes. Help us prevent the disappearance of democracy in Spain! Stop the German and Italian fascists from intervening in our country. The Republic of Spain is a legal government. We, the Communists, support and defend this government because it is the legitimate representative of the people who fight for democracy and liberty.

Dolores Ibarruri, Communist party leader

and orator, radio broadcast, July 1936

Document 9

It was not a revolutionary period leading to socialism. Not at all. That was just the pretext the Fascists used to justify their rising. The peasant masses weren‟t agitating for socialism. They wanted the Republic to tackle the country‟s fundamental problems, and land was one, if not the, major problem.

Narciso Julián, Communist railway worker,

recollection, July 1936

Documents 10-12: ANARCHISTS

Document 10

The Communists want to form a disciplined professional army to fight the Fascists. Militarization goes hand and hand with the hierarchical type of Communist organization. Such a professional army would lead to the creation of a state and any state is an oppressor.

Manuel Carabaño. worker and Anarchist

party youth member, 1936

Page 82: AP Euro DBQ Database

Document 11

It should be clearly understood that we are not fighting for the Democratic Republic. We are fighting for the triumph of the proletarian revolution. The revolution and the war are inseparable. Everything that is said to the contrary is reformist and counterrevolutionary.

Anarchist party information bulletin,

January 1937

Document 12

Two things are beginning to disappear: the privileges of class from which develops the monstrosity of war and the privilege that converts men into autonomous beings and women into slaves. Still, every day in Spain we hear talk of liberty for the oppressed, but we never hear these liberators refer to the necessity of declaring women free.

Free Women, an Anarchist

women‟s paper, 1937

Page 83: AP Euro DBQ Database

Document 14

The Basque nationalists were middle class in the main, religious and politically moderate. The Basque Nationalist party‟s decision to ally with the Communists and the Socialists, who stood for the destruction of the sort of society the Basques believed in, was the great drama of the Basque country; it created tremendous conflicts of conscience.

Juan Málzaga, Basque factory owner.

Recollection, Summer 1936

Page 84: AP Euro DBQ Database

Document 15

In Barcelona there are Catalonian nationalists; they are against anyone who opposes their cause.

Mariano Puente, merchant seaman

Recollection, 1936

Document 16

The Anarchists revolted against the Republican government in May 1937. The Republican forces and the Socialists, together with the Communists, began the counterattack. Fighting broke out all over Barcelona. The hate between the factions had risen to a feverish pitch.

Jaume Miravitlles, Catalonian Nationalist Liberal party, minister of information, diary, May 1937

Page 85: AP Euro DBQ Database

1991 DBQ

Directions: The following question is based on the accompanying Documents 1-15. (Some of the

documents have been edited for the purpose of this exercise.) Write your answer on the lined

pages of the pink essay booklet.

This question is designed to test your ability to work with historical documents. As you analyze

each document, take into account its source and the point of view of the author. Write an essay

on the following topic that integrates your analysis of the documents; in no case should

documents simply be cited and explained in a "laundry list" fashion. You may refer to historical

facts and developments not mentioned in the documents.

The Question: Analyze the views of those addressing the issue of slavery during the

Enlightenment and the French Revolution AND explain how those who debated this issue

thought its resolution would affect the economic, political and social order.

Historical Background. During the Enlightenment, French intellectuals addressed the institution

of slavery. then, at the time of the French Revolution, the National Assembly adopted the

Declaration of the Right of Man (August 27, 1789), which begins: "All men are born free and

remain free and equal in rights." The application of this statement raised issues concerning the

French colonies in the West Indies: Saint Domingue (now Haiti), Martinque, Guadeloupe,

Tobago, Saint Lucia and Saint Martin. French Merchants supplied these islands with slaves, and

French planters used the slaves to maintain their sugar and coffee plantations. The populations of

these colonies included African slaves, French colonists, mulatto landholders and freed Black

people. Slaves, mulattoes, and freed Black people had no political rights.

Page 86: AP Euro DBQ Database

THE EIGHTEENTH-CENTURY TRIANGULAR TRADE

Page 87: AP Euro DBQ Database

Document 1

Everything concurs to let humans enjoy dignity, which is natural. Everything tells us

that we can not take away from a person that natural dignity which is liberty.

Louis de Jaucourt, "The Slave," Encyclopedia, 1755

Document 2

One hundred thousand slaves, Black or mulatto, work in sugar mills, indigo and cocoa

plantations, sacrificing their lives to gratify our newly acquired appetites for sugar,

cocoa, coffee, and tobacco---things unknown to our ancestors.

Voltaire, Essay on Morals and Customs, 1756

Document 3

I have seen those vast unfortunate lands that seem only destined to be inhabited by

slaves. I have averted my eyes from that sordid sight with loathing, horror and pity; and

seeing one fourth of my fellow humans changed into beasts for the service of others, I

have grieved to be a human.

Jean-Jacques Rousseau, The New Heloise, 1761

Document 4

Why did the Christian powers not consider that their religion, independent of natural

law, was fundamentally opposed to Black slavery? The answer is that those nations

needed slaves for their colonies, their plantations, and their mines.

Denis Diderot, "Natural Liberty," Encyclopedia, 1765

Document 5

Masters who acquired new slaves were obligated by law to have them instructed in the

Catholic faith. this motivated Louis XIII to authorize this horrid commerce in human

flesh.

Louis de Jaucourt, "Blacks," Encyclopedia, 1765

Page 88: AP Euro DBQ Database

Document 6

White people are incapable of working in the field under the hot sun in Saint Dominque;

thus to make the best of this precious soil, it has been necessary to find a particular

species of laborers. Saint Domingue is a milder climate for the slaves than the hot

climate from which they have been transplanted.

Guillaume Raynal, Essay on the Administration of Saint Domingue, 1781

Document 7

Why are Black people enslaved? The color of people's skin only suggests a slight

difference. There is no discord between day and night, the sun and the moon and

between the stars and dark sky. All is varied; it is the beauty of nature. Why destroy

nature's work?

Olympe de Gouges, Reflections on Black People, 1788

Document 8

A day may come, gentlemen, when you will cast a glance of compassion on these

unfortunate people who have been made a barbaric object of trade; these people who are

similar to us, in thought and, above all, in their capacity to suffer.

Jacques Necker, speech, opening meeting of the Estates General, May 1789

Document 9

I demand to know how the twenty White people here from the colonies can be said to

represent the people of color from whom they have received no authority. I demand to

know by what right the 23,000 White voters have refused their fellow citizens the right

to name representatives and have arrogated to themselves the right to choose

representatives for those whom they have excluded.

Count Mirabeau, speech, National Assembly, July 3, 1789

Document 10

The abolition of slavery and the slave trade would mean the loss of our colonies; the loss

of the colonies would strike a mortal blow to commerce, and the ruin of commerce

would result in stagnation for the merchant marine, agriculture, and the arts. Five million

French citizens exist only by the trade they bring. The colonies bring in an annual

income of more than 200 million livres.

A delegate from Bordeaux, speech, National Assembly, March 2, 1790

Page 89: AP Euro DBQ Database

Document 11

SLAVES DELIVERED BY FRENCH SHIPS

Document 12

Seems to me that it would be possible to conciliate the interest of commerce, that of the

colonies, and that of all France; and for that I propose to name a committee which will be

sent all the papers relative to Saint Domingue and to Martinque. this committee will

present to you, in a few days, a definite plan of procedure. We will gain time.

Charles de Lameth, speech, National Assembly, early 1790

Page 90: AP Euro DBQ Database

Document 13

End our fears by declaring that your proclamation on the Rights of Man does not extend

to the Black people and their descendants. We have not enslaved them, but we

discovered them in the hardest and cruelest slavery, and transplanted them to French

colonies, under a kind of humane government, where, indeed, they work, but they live

without fear for tomorrow.

A delegate of the Owners of Property in the French Colonies of America Residing in

Bordeaux, speech, National Assembly, date unknown

Document 14

We have reached this level of prosperity thanks to our colonies. If someday they must

gain independence, we must make sure to postpone that day so that we will be able to

lose them without an economic shock and without a disturbance to our political

existence.

Antoine Barnave, report, National Assembly's Committee on the Colonies, 1790

Document 15

I am here to defend the Declaration of the Rights of Man. Let the colonies perish if the

planters, with their threats, try to force us to legislate in their private interest! I declare in

the name of the Assembly, in the name of those members of the Assembly who do not

want to destroy the Constitution, I declare in the name of the entire nation which wishes

to be free, that we will not sacrifice to the colonial deputies. I say that any other course,

whatever it might be, is preferable.

Maximilien Robespierre, National Assembly, May 15,, 1791

Page 91: AP Euro DBQ Database

1992

Directions: The following question is based on the accompanying Documents 1-14. (Some of the documents have been edited for the purpose of this exercise.) Write your answer on the lined pages of the pink essay booklet. This question is designed to test your ability to work with historical documents. As you analyze the documents, take into account the source and the point of view of the author. Write an essay on the following topic that integrates your analysis of the documents; in no case should documents simply be cited and explained in a “laundry list” fashion. You may refer to historical facts and developments not mentioned in the documents.

The Question: Identify and analyze the political and cultural issues in the debate

over Pan-Slavism

Historical Background: In the nineteenth century most Slavic peoples lived in multinational empires in eastern and southern Europe, where growing nationalism and international rivalries for control over territories and peoples persisted throughout the century. Pan-Slavism, a movement intended to promote the unity of all Slavic peoples, began to emerge in the early nineteenth century; the movement experienced a surge of popularity and activity beginning in 1848.

Page 92: AP Euro DBQ Database

Document 1.

Page 93: AP Euro DBQ Database
Page 94: AP Euro DBQ Database

Document 3

In other nations humanity comes after nationality. Among the Slavs nationality comes after humanity. Scattered Slavs, let us be a unified whole, and no longer mere fragments. Let us be all or nothing. Who are you, a Russian? And you, a Serb? And you, a Czech? And you, a Pole? My children, seek unity! Say: I am a Slav!

Jan Kollár, Slovak poet and early advocate of Pan-

Slavism, 1829

Document 4

There are Slavic languages, literatures, and there are also several Slavic homelands. Due to ignorance of each other, hatred, oppression, and differences of language, there is today neither a single Slavic language nor a common Slavic literature nor a Slavic homeland. To put the Polish national cause under the protection of a Slavic idea, which in reality does not exist, would lead Poland astray.

Karol Sienkiewicz, Polish author, introduction to a

collection of historical studies on Slavs, 1842

Document 5

There is only one way for Austria to forestall the penetration of Russian influence among the western and southern Slavs --- Austria must put itself at the head of the Slavs and promote their national development. At one stroke Austria will thus destroy all illusions of a Russian Pan-Slavism and will find a firm and unshakable support among its own peoples, who would no longer regard Austria as an alien ruler.

An editorial in Contemporary Austrian Review, 1843

I

Page 95: AP Euro DBQ Database

Document 6

At the present time there is no place for your Pan-Slavic goals in the Russian empire. You ask for life, and there is only the silence of death; you demand independence, and in Russia there is only mechanical obedience; you aspire toward resurrection, uplift, right, and liberty, and there are only death, darkness, and slave labor.

Mikhail Bakunin, Russian anarchist, speech at the

Pan-Slav Congress, Prague, 1848

Document 7

I express my firm conviction that the Slavs, I mean the Russians, the Poles, the Czechs, etc., are not one nation. The name Slav is and should forever remain a purely geographical name. Nationality is not only determined by language, but also by customs, religion, form of government, state of education, sympathies, etc.

Karel Havlkek, Czech journalist, article entitled “Slav and Czech,” 1848

Document 8

If I were tsar today, then I would form a free and happy Pan-Slav Empire. I would first renew an independent Poland, thereby winning the hearts of the Czechs, the Serbs, and all the southern Slavs. Everywhere I would plant the banner of liberty. I would destroy, without effort, the Ottoman and the Austrian empires. The Slavs would rush into this battle in great number.

Bronislaw Trentowski, Polish philosopher, lecture, Cracow, 1848

Document 9

The idea of Slavic unity appeared only recently among the southern Slavs, and has developed little because it ignores our distinct identities. The Prussian is a German, the Piedmontese is an Italian, but a Bulgarian is not a Serb and a Serb is not a Russian. A small federation of southern Slavs should be built so that no nationality may be wronged.

Christo Boter, Bulgarian poet, 1867

Page 96: AP Euro DBQ Database

Document 10

A great and strong Russia has to face the difficult task of liberating its racial brothers; for this, Russia must steel them and itself in the spirit of independence and Pan-Slavic consciousness.

Nikolay Danilevsky, Russian, Russia and Europe: An inquiry into the Cultural and Political Relations of the Slav World and the Germano-Roman World, 1869

Document 11

The Pan-Slav party in Russia seeks to absorb and to destroy our nationality. I must therefore reject in advance every idea of a Czech republic or any other republic within the present boundaries of the Austrian empire. Think of the Austrian empire divided up into a number of republics. What a delightful invitation for the Russians to create a universal monarchy!

Frantisek Palacky, Czech, article in the

Viennese weekly The Reform, 1873

Document 12

The danger that Austria has to face is the diversity of language and race in the empire. Our Slavic nationalities are likely at a moment of dangerous crisis to develop pro-Russian tendencies.

Count Friedrich Ferdinand von Beust, Austrian foreign minister

and imperial chancellor (1866-1881), Memoirs, 1887

Document 13

Russia is united by faith, blood, and historical tradition with the Slavic peoples and has never regarded their fate with indifference. The fraternal feelings of the Russian peoples for the Slavs were aroused in unanimous enthusiasm and with special force when Austria-Hungary put before Serbia conditions manifestly unacceptable to a sovereign state.

Tsar Nicholas II, Manifesto, 1914

Page 97: AP Euro DBQ Database

Document 14

No Russian ever wanted the reunion of other Slavs with Russia, and no agitation in that sense has ever been practiced in Slavic lands by the Russians. Pan-Slavism in a theoretical sense existed only among the weakest and most oppressed Slavic peoples. These people used to visit Russia to complain of their sufferings and persecutions, and tried unsuccessfully to excite Russian sympathy.

Gabriel de Wesselitsky, Russian journalist. Russia and Democracy:

The German Canker in Russia, 1915

Page 98: AP Euro DBQ Database

1993 DBQ

Directions: The following question is based on the accompanying Documents 1-13. (Some of the documents have been edited for the purpose of this exercise.) Write your answer on the lined pages of the pink essay booklet. This question is designed to test your ability to work with historical documents. As you analyze the documents, take into account both the sources and the authors‟ points of view. Write an essay on the following topic that integrates your analysis of the documents. In no case should documents simply be cited and explained in a “laundry list” fashion. Do not simply summarize the documents individually. In your analysis of the documents, you may refer to relevant historical facts and developments not mentioned in the documents.

The Question: Describe the various values and purposes of Renaissance education. Analyze the

extent to which these values and purposes were transformed and challenged over time.

Document 1

So far we have touched upon studies (grammar, rhetoric, geometry, music) by which we may

attain enlightenment of the mind. However, we have not yet directly considered how we may

most surely distinguish the true and the just from the base and degrading. Need I then impress

upon you the importance of the study of Philosophy and of Letters…our guide to the true

meaning of the past, to a right estimate of the present, to a sound forecast of the future. Where

Letters cease, darkness covers the land; and a Prince who cannot read the lessons of history is a

helpless prey of flattery and intrigue.

Aeneas Sylvius Piccolomini, Italian humanist

who later became pope, On the Education of Free Men, 1450

Document 2

Learning and training in Virtue, which the ancients called the “Humanities,” are peculiar to man,

for they are the pursuits and the activities proper to mankind.

Battista Guarino, Italian humanist educator,

On the Method of Teaching and Learning,

1459

Page 99: AP Euro DBQ Database

Document 3

The courtier should be passably learned in the humanities, in the Latin poets, orators, and

historians, and should also be practiced in writing verse and prose, especially in our own

vernacular. In this way he will never want for pleasant entertainment with the ladies, who are

usually fond of such things and even if his writings should not merit great praise, at least he will

be capable of judging the writing of others.

Baldassare Castiglione, Italian diplomat and author,

The Book of the Courtier, 1528

Document 4

When once the simpler rules of composition, in prose and verse, and the commoner figures of

speech have been mastered, the whole stress of teaching must be laid upon a close yet wide study

of the greater writers. The student devotes his attention to the content of the literatures of ancient

Greece and Rome because with slight qualification the whole of attainable knowledge lies

therein.

Desiderius Erasmus, northern humanist and theologian,

On the Art of Learning, 1511

Document 5

Learned women may be suspected by many who say learning is a nourishment for the

maliciousness of their nature. When a woman is taught to read the classics, let the books teach

her good manners. And when she learns to write, let not her example be trifling songs but some

sober sentences, prudent and chaste, taken out of holy scripture or the sayings of philosophers.

Juan Luis Vives, Spanish humanist,

The Instruction of a Christian Woman, 1523

Document 6

When I was young, I made light of good penmanship, knowing how to ride, play, dance and sing

and dress well, which are things that seem more decorative than substantial in a man. But later, I

wished I had not done so. For although it is not wise to spend too much time cultivating these

arts. I have seen that they lend dignity and reputation even to men of good rank. Skills of this

sort open the way to the favor of princes, and sometimes to great profit and honors.

Francesco Guicciardini, Italian Statesman and historian,

Reflections, 1530

Page 100: AP Euro DBQ Database

Document 7

At least twice a year, each pastor should admonish his parishioners that they be diligent in

sending their children to school, not only for learning the liberal arts, but also the fear of God,

virtue, and discipline. Otherwise, permanent harm must result, as children grow up without fear

and knowledge of God, without discipline, learning nothing about what is needed for their

salvation, nor what is useful to them in worldly life.

From the School Ordinances of Wurttemberg, Germany, 1559

Document 8

The aim of our absurd educational system has been to make us, not good and wise, but learned;

and it has succeeded. It has selected, for our instruction, not those books which contain the

soundest and truest opinions, but those which speak the best Greek and Latin.

Michael de Montaigne, French essayist and politician,

“Of Presumption,” 1578-1580

Document 9

Let me recommend the gentlewoman whose school we spoke of: she teaches girls embroidery,

reading, writing, and dancing; for music you must pay extra. She has teachers for singing and

playing instruments.

Anne Higginson, Letter to Lady Ferrers of Tamworth

Castle, England, late sixteenth century

Document 10

It is notorious that, in most of our common schools, the scholars at fifteen or sixteen years of age

have little sense of the meaning and true use of learning, but can only write Latin no one of

judgment will want to read. When they go to the universities, they waste their friends‟ money

and their own precious time. Afterwards, they return home again, almost as crude as when they

went.

John Brinsley, English schoolmaster, A Consolation for our

Grammar Schools, 1622

Page 101: AP Euro DBQ Database

Document 11

In general, it can be said that schools are useful in a civilized society, but having too many of

them is always a bad thing. The study of literature is appropriate only to a small minority of

men. Such study weakens the body and inspires contempt for all other occupations. More

farmers are needed than magistrates, more soldiers than priests, more merchants than

philosophers, more hard-working bodies than dreamy and contemplative spirits.

Letter to the Parlement of Dijon concerning the reopening of a

French Jesuit school, mid-seventeenth century

Document 12

Percentage of Justices of the Peace Who Attended University (by county)

Date

Kent Northhamtonshire Somerset

1562 2 % 6 % 3 %

1584 16 17 15

1608 40 19 36

1626 63 54 50

1636 68 72 55

Document 13

Learning is not enough accommodated to the uses of our life, to teach us how to behave

ourselves in the occurrences thereof. The fault whereof must be laid upon that inveterate

custom, or rather disease of Schools, whereby all the time of youth is spent in Grammatical,

Rhetorical, and Logical toys; those things which are real, and fit to enlighten men‟s minds, and

to prepare them for action, being reserved for the Universities.

John Amos Comenius, educational reformer in Bohemia,

A Reformation of Schools, 1642

Page 102: AP Euro DBQ Database

1998 DBQ

Directions: The following question is based on the accompanying Documents 1-11. (Some of the documents have been edited for the purpose of this exercise.) Write your answer on the lined pages of the pink essay booklet. This question is designed to test your ability to work with historical documents. As you analyze the documents, take into account both the sources and the authors‟ points of view. Write an essay on the following topic that integrates your analysis of the documents. In no case should documents simply be cited and explained in a “laundry list” fashion. Do not simply summarize the documents individually. In your analysis of the documents, you may refer to relevant historical facts and developments not mentioned in the documents.

The Question: Describe and analyze the views of those who were concerned about the problems of the political, economic, and social order in the German states before the revolutions of 1848.

Historical background: In the eighteenth century, the Germans were divided among more than three hundred states, ranging from great powers (Austria and Prussia) to small city-states and principalities, all grouped under the Holy Roman Empire. During the Napoleonic Wars, some Germans hoped for German unification under a single constitutional monarchy. In 1815, after the defeat of Napoleon, much of the previous social and political order was restored. There were thirty-eight states loosely tied together in the Germanic Confederation. The map on the following page shows the major states of that confederation.

Page 103: AP Euro DBQ Database
Page 104: AP Euro DBQ Database

Document 1 Source: Ernst Moritz Arndt, German poet and professor, 1813.

Where is the German‟s Fatherland? Is it Bavaria? Is it the Prussian-land?

Is it Tyrol? Is it where the Swiss dwell? Ah! Austria surely it must be? Oh no! More great, more grand

Must be the German‟s Fatherland!

Where is the German‟s Fatherland? Wherever resounds the German tongue!

Document 2 Source: Joseph von* Goerres, German publicist and scholar in exile in France, pamphlet entitled “Germany and the Revolution,” 1819. In Germany I am pleased a new idea is added to those that caused the revolution in France — the idea of national unity, which will render the ferment stronger than ever. A German revolution must end with the expulsion of the reigning princes, the overthrow of all ecclesiastical establishments, the destruction of the nobles, and the introduction of a republican constitution. *The term “von” is a sign of aristocratic status.

Document 3 Source: Klemens von Metternich, Austrian chief minister, memorandum to the Austrian emperor, 1819. Formerly the German revolutionaries were separated by the states in which they lived. It was clear to those conspirators that under such circumstances they could strike no effective blow. Some of these men now take the correct road from a revolutionary point of view. They direct their eyes to the union of all Germans in one Germany. This evil idea must be conquered.

Page 105: AP Euro DBQ Database

Document 4

Source: David Hansemann, an industrialist in the Prussian Rhineland, private letter, 1830.

We liberals insist that no one suffer distress, and so one institution after another is founded to feed the poor, to educate their children, to care for the old, to help poor mothers, etc. But herein lies the most direct invitation to wastefulness and laziness, the two vices which will most effectively nourish good-for-nothings among the lower classes. These good-for-nothings are dangerous to the public safety.

Document 5

Source: Friedrich List, Wurttemberg economist and academic, pamphlet, 1834.

Thirty-eight customs borders dividing the German states cripple our internal commerce and bring about the same effect as binding up every part of the human body so that blood cannot flow from one to the other.

Document 6

Source: Essay by Johann Riegel, a bookseller in a university town in Wurttemberg, 1842.

We live in a transitional period. Factories are taking the place of craft production. Nearly all the crafts are either in decline or in the grip of drastic changes in their shops to meet the competition of industrialization.

Document 7 Source: Bettina von Arnim, author and wife of a Prussian aristocrat, This is the Responsibility of the King, book dedicated to King Frederick William IV of Prussia, 1843. The father weaves cloth for jackets and pants, but he himself is in rags. The children are naked, trying to warm themselves in the straw. The mother spins threads from daylight to dark, but her efforts can never satisfy the needs of her children. The state demands taxes from the family, and they must pay their rent or the landlord will evict them.

Page 106: AP Euro DBQ Database

Document 8 Source: General Joseph von Radowitz, advisor to King Frederick William IV of Prussia, Concerning State and Church, book, 1846. Our German princes still have the resources to survive the struggle against the triumphant mediocrity of the middle classes. Let our princes have the courage to turn to the masses. There, among the lower and most numerous classes of the population, are their natural allies. The bourgeoisie has been corrupted by the evil education of the times and has lost its loyalty and faith.

Page 107: AP Euro DBQ Database

Document 9

Page 108: AP Euro DBQ Database

Document 10 Source: Hans von Gagern, government official in Hesse, speech before the Hessian State Assembly, 1847.

A new spirit is now irresistibly asserting itself in Germany. It is a strengthened public spirit, and in our times the German people cannot be put off as they were in previous years. It is the unquestionable conviction of the whole people that only by developing the principle of a representative and constitutional monarchy throughout Germany can the unity of the fatherland be strengthened, freedom come forth, and the rule of law be secured for our future public life.

Document 11 Source: Anonymous pamphlet confiscated by the police in Frankfurt, 1847. Men of the Proletariat! German workers! You are the heart of the people. Show what you are worth. It is an honor to be called “the proletariat.‟ Be worthy of this honor, and show that you were not born to be hunted like wild animals by the prince‟s police. When it comes time to fight— attack!